Study Guide / Text Questions (Chapter 3, 4,

Pataasin ang iyong marka sa homework at exams ngayon gamit ang Quizwiz!

Ulcerative colitis is found in the a. Large intestine b. Small intestine c. Stomach d. Anywhere in the GI tract from the mouth to anus

a. Large intestine

Treatment for tinea unguium must include? a. Oral antifungals b. Topical Antifungals c. Both oral and topical d. Nail removal and topical antifungals

a. Oral antifungals

Which of the following clinical manifestations is not typically found with inflammation but is more characteristic of a bacterial infection? a. Purulent exudate b. Redness and swelling at the site c. Lymphadenopathy d. Fever

a. Purulent exudate

A 31 year-old man who has worked for several years installing fiberglass insulation has developed itchy, irritating lesions on his wrists and forearms over the last several months. He has applied moisturizing creams repeatedly and has taken antihistamines but has experienced no significant improvement. He is understandably concerned about the potential effect this could have on his livelihood and has asked his care provider when treatment will resolve the problem. What is the care provider's most appropriate response? a) "You're clearly allergic to the fiberglass in your insulation, and over-the-counter allergy medications are likely to resolve the problem quite rapidly." b) "The problem is that your skin is producing and sloughing off cells prematurely; this will respond well to a topical steroid." c) "There are medications I can prescribe that will cure this sensitivity, but they tend to take many months to take full effect." d) "A steroid cream will likely help, but often the skin problem lasts long after contact with the product that irritates the skin."

"A steroid cream will likely help, but often the skin problem lasts long after contact with the product that irritates the skin." Topical corticosteroids are often used in the treatment of irritant contact dermatitis, but symptoms can persist long after contact with the irritant ceases. The course of the man's complaint is suggestive of irritant, not allergic, contact dermatitis. Hyperkeratinization is characteristic of psoriasis and a sensitivity can be treated but not cured.

A patient recently diagnosed with rheumatoid arthritis (RA) tells the nurse she is glad there is nothing "really wrong with her" but some joint swelling. Which of the following information should the nurse tell the patient about RA? a) "Myocardial infarction develops in clients who have had the disease for an extended period." b) "Extra-articular manifestations may include anemia and deformities of the affected joints." c) "Joint involvement is typically the only manifestation of the disease in most clients." d) "The disease can be reversed with medication and exercise."

"Extra-articular manifestations may include anemia and deformities of the affected joints." Although characteristically a joint disease, RA can affect a number of other tissues including red blood cells, blood vessels, lungs, and heart. Treatment goals for a person with RA are to prevent and/or reduce the pain, decrease stiffness and swelling, maximize mobility, and possibly halt the progression, but not cure the disease.

A patient has been diagnosed with rheumatoid arthritis (RA). Which of the following will the nurse tell the patient about this disorder's etiology? a) "Environment is the biggest contributing factor to the development of RA." b) "Exposure to workplace chemicals is a causative agent." c) "Genetic predisposition is very likely." d) "The disease is most common in those under 30."

"Genetic predisposition is very likely." The cause of RA is uncertain but evidence points to genetic predisposition. The disease usually occurs later in life.

A patient was diagnosed with human immunodeficiency virus (HIV), which causes acquired immunodeficiency syndrome (AIDS). The nurse realizes that the patient needs further teaching about the disease process based on which of the following statements made by the patient? a) "The infection requires a host cell." b) "After infection the HIV virus can exist in a latent state." c) "HIV is a retrovirus." d) "HIV only infects B cells."

"HIV only infects B cells." The retroviruses such as HIV have a unique mechanism of replication. After entry into the host cell, the viral RNA genome is first translated into DNA by a viral enzyme called reverse transcriptase. The viral DNA copy is then integrated into the host chromosome where it exists in a latent state, similar to the herpes viruses. Reactivation and replication require a reversal of the entire process. Some retroviruses lyse the host cell during the process of replication. In the case of HIV, the infected cells regulate the immunologic defense system of the host, and their lysis leads to a permanent suppression of the immune response.

A client presents with an exacerbation of her psoriasis. The most important question for the nurse to ask the client would be: a) "Have you traveled to a foreign country in the last 14 days?" b) "Have you recently been sexually active?" c) "Have you been under an excessive amount of stress at home or at work?" d) "Have you been eating large amounts of oily foods?"

"Have you been under an excessive amount of stress at home or at work?" Environmental stimuli may trigger the release of cytokines and growth factors from keratinocytes and other cells, with ensuing immune and inflammatory responses that lead to the full development of psoriatic lesions. Psoriatic lesions can be induced in susceptible individuals by trauma, a process known as the Köbner phenomenon. Precipitating factors for an exacerbation of psoriasis include increased stress, infection, trauma, and the use of certain medications. Types of food ingested, sexual activity, and travel to foreign countries have not been linked to psoriasis.

A nurse is teaching a group of nursing students about the presentation of systemic lupus erythematosus (SLE). Which of the following statements does the nurse is likely to make? a) "Drug-induced lupus requires lifelong treatment." b) "More women than men are affected by lupus." c) "Lupus is more common in Caucasians." d) "Lupus is a disease of older individuals."

"More women than men are affected by lupus." There is a female predominance of 10:1 in those with SLE. This ratio is closer to 30:1 during childbearing years. SLE is more common in African Americans, Hispanics, and Asians than in whites, and the incidence in some families is higher than in others.

Following a progressive onset of fatigue, aching and joint stiffness over the last two years, a 69 year-old male has recently been diagnosed with rheumatoid arthritis (RA). Which of the following teaching points should his primary care physician include during the office visit in which this diagnosis is communicated to the client? a) "Steroids and anti-inflammatory drugs that I'll prescribe will likely bring some relief to your symptoms." b) "The symptoms you've been experiencing are the result of damage inside your joints, but I'll start you medications that will reverse this damage." c) "The best treatment plan is to try all other available treatments before resorting to using medications." d) "It's important that you maximize your level of activity, since decreasing your mobility will worsen the disease."

"Steroids and anti-inflammatory drugs that I'll prescribe will likely bring some relief to your symptoms." Current treatment guidelines for RA involve early and aggressive pharmacological treatment, including NSAIDs and corticosteroids. Damage cannot be reversed, and while therapeutic exercise plays a role in treatment, rest is also important.

A client tells the health care provider he has heard several myths about viruses. The most appropriate information for the nurse to provide would be: a) "Viruses are capable of replicating outside of a living cell." b) "Viruses kill cells they come in contact with immediately." c) "Viruses do not require a host cell to replicate." d) "Viruses can produce symptoms of disease months to years later."

"Viruses can produce symptoms of disease months to years later." Viruses can produce symptoms of disease month to years later. Some viruses, such as hepatitis C, can remain latent for long periods of time without causing disease until months to years later. Viruses are incapable of replicating outside of a living cell and are incapable of replication unless in a host cell. Not every virus causes lysis and death of the host cell during replication.

The nurse is caring for a client who has just learned that he is HIV positive. The client asks the nurse how long he has been able to infect others. Select the best response by the nurse. a) "You can infect others when you start to get symptoms." b) "You can infect others before the HIV antibody is detectable in your blood." c) "You can infect others only after you have experienced seroconversion." d) "You can infect others only when you are actually diagnosed with AIDS."

"You can infect others before the HIV antibody is detectable in your blood." The HIV-infected person is infectious even when no symptoms are present. The point at which an infected person converts from being negative for presence of HIV antibodies in the blood to being positive is called seroconversion. The time after infection and before seroconversion is known as the window period. During the window period, a person's HIV antibody test result will be negative, but the infected person may still transmit the virus.

Which of the following teaching points is most appropriate for a teenager who has sought care for the treatment of his severe acne? a) "All the creams and ointments that you can buy have been shown to be no real help for acne." b) "You might need antibiotic pills in addition to a cream for your face." c) "It's important for you to vigorously wash your face several times a day." d) "Avoiding high-fat foods and chocolate won't cure your acne, but it will likely improve it a lot."

"You might need antibiotic pills in addition to a cream for your face." Combination treatments for acne often include a topical preparation in addition to oral antibiotics. Avoiding certain foods has not been shown to be effective in acne treatment or prevention and the topical products available are effective in some, though not all, cases. Care should be taken not to exacerbate lesions by washing the face too vigorously or too often.

Shortly after being diagnosed with HIV, a client has begun highly active antiretroviral therapy (HAART). The client asks, "My doctor tells me that my viral load is going down. What does that mean?" The nurse's best response is: a) "Your HAART medications are working to slow the progression of the disease." b) "Your medications are going to decrease your ability to transmit the virus to your sexual partners." c) "You are developing drug resistance and may need to have your medications adjusted." d) "This means that you are in the long-term nonprogressors stage of HIV."

"Your HAART medications are working to slow the progression of the disease." Antiviral therapies such as HAART are prescribed to slow the progression of AIDS and improve the overall quality of life and survival time of persons with HIV infection. Extension, not limitation, of the latent period is a goal. Minimizing transmission and preventing seroconversion are not normally achievable goals through drug therapy alone. The final 5% to 15% are long-term nonprogressors, who remain asymptomatic for 10 years or more after seroconversion, with stable CD4+ T-cell counts and low plasma HIV RNA levels.

A 16-year-old male is concerned about the acne that has developed on his face and asks the nurse what caused it to develop. The best response would be: a) "You are not scrubbing your face hard enough to prevent the acne from occurring." b) "Your acne has developed because of increased hormone levels in your body." c) "If you keep your hands off your face when sitting, it will completely clear." d) "You have acne because you eat a lot of fatty, oily foods."

"Your acne has developed because of increased hormone levels in your body." Acne typically occurs in the teen years related to increased androgen hormones, which increase the activity of the sebaceous cells. Food has not been found to cause acne as once thought. Scrubbing the face may actually increase the acne.

The nurse is caring for a client who is a carrier of hepatitis B. Which of these does the nurse teach the family? a) "Your loved one may not look ill, but the virus is present in his blood." b) "The tests your loved one took show he has had hepatitis B in the past." c) "A carrier state means the individual is at risk for sudden death." d) "Something in your home causes him to 'carry' the virus to his body."

"Your loved one may not look ill, but the virus is present in his blood." Infection with HBV and HCV can produce a carrier state in which the person does not have symptoms but harbors the virus and can therefore transmit the disease.

Why is Crohn disease more likely to cause intestinal obstruction than ulcerative colitis? a. Crohn disease is located in the small intestine b. Crohn disease causes granulomas to form in the submucosal layer c. Crohn disease causes abdominal pain and watery diarrhea d. Crohn disease is exacerbated by certain foods, such as spicy foods

. Crohn disease causes granulomas to form in the submucosal layer

A patient is being seen in the primary care physician office for a follow up appointment for a second degree burn. The burn is healing. In approximately what time frame do second degree burns maintain their softness and elasticity but may have loss of sensation with scar formation? a) 4 months b) 1 month c) 3 months d) 2 months

1 month After healing, in approximately 1 month, these burns maintain their softness and elasticity, but there may be the loss of some sensation. Scar formation is usual.

Place the following in the sequence of the progression of HIV/AIDS on a cellular level. 1)The HIV virus is uncoded. 2)HIV attaches to the receptors on the CD4+ cell. 3)DNA synthesis occurs by reverse transcription. 4)Transcription of the DNA to form a single-strand messenger RNA. a) 1,2,3,4 b) 4,3,1,2 c) 2,1,3,4 d) 3,4,1,2,

2,1,3,4 The first step involves the binding of the virus to the CD4+ T cell. The second step allows for the internalization of the virus. After attachment, the virus envelope peptides fuse to the CD4+ t-cell membrane. Fusion results in uncoding of the virus. The third step consists of DNA synthesis by reverse transcription. The fourth step is integration. The fifth step involves transcription of the double-stranded viral CNA to form a singe-stranded messenger RNA.

Which of the following types of viral hepatitis is not associated with transmission through contact with infected blood? a) D b) A c) B d) C

A Infections can be transmitted from person to person through shared inanimate objects contaminated with infected body fluids (fomites). An example of this mechanism of transmission would include the spread of the HIV and hepatitis B, C and D virus through the use of shared syringes by intravenous drug users. Hepatitis A is acquired through the injection of contaminated food or fluids.

Which of the following patients is likely at the greatest risk of developing a urinary tract infection? a) A patient with a diagnosis of chronic kidney disease who requires regular hemodialysis b) A pregnant woman who has been experiencing urinary frequency c) A confused, 81-year-old patient who is incontinent of urine d) A 79-year-old patient with an indwelling catheter

A 79-year-old patient with an indwelling catheter Indwelling catheters are strongly associated with the development of UTIs, and this risk factor supersedes pregnancy and kidney disease. Frequency and incontinence may be signs and symptoms of UTIs, but they are not causative of the infections.

Which of the following individuals is at the highest risk of developing a urinary tract infection (UTI)? a) A 60 year-old man with a history of cardiovascular disease who is recovering in hospital from a coronary artery bypass graft. b) A 38 year-old man with high urine output due to antidiuretic hormone insufficiency. c) A 66 year-old man undergoing dialysis for the treatment of chronic renal failure secondary to hypertension. d) A 30 year-old woman with poorly controlled diabetes mellitus.

A 30 year-old woman with poorly controlled diabetes mellitus. Young women as well as persons with diabetes are at high risk of UTIs. Neither post-surgical recovery nor renal failure is necessarily direct risks for UTI development, and high urine output would prevent decrease rather than increase UTI risk.

Which of the following wounds is most likely to heal by secondary intention? a) A needlestick injury that a nurse received when injecting a patient's medication b) A boy's "road rash" that he got by falling off his bicycle c) A finger laceration that a cook received while cutting up onions d) The incision from a teenager's open appendectomy

A boy's "road rash" that he got by falling off his bicycle Incisions, cuts, and puncture wounds are likely to heal by primary intention, whereas larger wounds with loss of tissue and contamination (such as may be experienced in a bicycle crash) are likely to heal by secondary intention.

While explaining to a post-surgical knee client about the various forms and function of connective tissue, the nurse gives the example of the client's surgical repair of a torn anterior cruciate ligament (ACL), which is due to: a) A dense regular connective tissue tear that is usually rich in collagen fibers that allows ligaments to join bone to bone b) Irregular filling of spaces between tissues to facilitate keeping of joints and tendons in their proper place c) A reticular fiber problem that interrupted the framework for capillaries d) An irregular, dense connective tissue tear of loose connective tissue that is located in the perichondrium

A dense regular connective tissue tear that is usually rich in collagen fibers that allows ligaments to join bone to bone Dense regular connective tissues are rich in collagen fibers and form the tendons and aponeuroses that join muscles to bone or other muscles and the ligaments that join bone to bone. Dense irregular connective tissue consists of the same components found in loose connective tissue but exhibits a predominance of collagen fibers and fewer cells. This type of tissue can be found in the fibrous sheaths of cartilage (i.e., perichondrium) and bone (i.e., periosteum). Fibroblasts, the most abundant loose connective tissue cells, synthesize the gel-like substance and collagen, elastin, and reticular fibers. Reticular fibers provide a fibrous framework for capillaries. Adipose tissue helps to fill spaces between tissues and keep organs in place.

Mona then asks you how the antibodies that Tyra develops will protect her from future encounters with the varicella zoster virus. You tell her that antibodies protect the body in all of the following ways except: A. Assist with the binding of the antigen to the antibody B. Help promote phagocytosis and destruction of the pathogen C. Assist with activation of complement D. Help to bind the neutrophils to the T cells

A. Assist with the binding of the antigen to the antibody

Donna's exposure to the cold virus occurred when she was with Alexia. However, Donna did not experience symptoms of a cold right away. This period of time from the time of exposure until Donna developed symptoms is called the: A. Incubation period B. Exposure period C. Prodromal period D. Convalescence period

A. Incubation period

Teri's brother accidentally hooked her arm with a fish hook while they were fishing for sunfish off of their grandparents; dock. Her grandfather removed the hook and cleansed the are with soap and water. One week later, an infection developed and the tissue around the wound was red, painful, swollen, and warm to touch. The redness, pain swelling, and warmth in the tissue are due to the: A. Inflammation process B. Bactercidal process C. Septic shock D. Chronic infection

A. Inflammation process

Cell mediated immunity protects the body in all of the following ways except: A. It decreases antigody production B. It activates macrophages C. It activates antigen specific cytotoxic T lymphocytes D. It stimulates the production of cytokines

A. It decreases antigody production

Your friend Mona has a 4 year old sister named Tyra. She tells you that over the weekend the mother of one of Tyra's friends called to say that her daughter had the chicken pox (varicella zoster virus). If Tyra develops chicken pox, what type of immunity will he develop? A. Naturally acquired active immunity B. Naturally acquired passive immunity C. Artificially acquired active immunity D. Artificially acquired passive immunity

A. Naturally acquired active immunity

Excherichia coli is present within the gastrointestinal tract and does not pose any harm to the body. This type of bacteria is an example of: A. Resident flora B. Anaerobic flora C. An overproliferation of bacteria D. A fungal infection

A. Resident flora

Shana has scheduled a visit with a fertility clinic because she is unable to become pregnant. When taking her history, it is noted that she had been diagnosed with numverous sexually transmitted infections in the past. Shana wonders how those infections in the past could possibly affect her ability to become pregnant now. You know that repeated or chronic infections will: A. Result in scar tissue formation within the fallopian tubes B. Enhance ovulation C. Decrease the risk for future sexually transmitted infection D. Increase the number of eggs released from the fallopian tubes

A. Result in scar tissue formation within the fallopian tubes

These cells display antigens for recognition by the cytotoxic T lymphocytes: A. T helper lymphocytes B. Neutrophils C. Granulocytes D. Eosinophils

A. T helper lymphocytes

Becky has an infection and her white blood cell count indicates leukopenia. Which of the following is most likely to be occurring? A. Viral infection B. Bacterial infection C. Fungal infection D. Adequate immune response

A. Viral infection

Which of the following terms relates to potency, and therefore the pathogen;s ability to cause disease, in large populations even with minimal exposure? A. Virulence B. Antigenicity C. Infectivity D. Toxigenicity

A. Virulence

A distinguishing feature of viral influenza is: a) Direct contact transmission b) Profuse watery nasal discharge c) Abrupt-onset, profound malaise d) Constant pounding headache

Abrupt-onset, profound malaise One distinguishing feature of influenza is the rapid onset, sometimes within minutes, of profound malaise. As with many viral respiratory tract infections, transmission is by aerosol or direct contact. In the early stages, the symptoms of influenza often are indistinguishable from other viral infections—fever, chills, malaise, muscle aching, headache, profuse watery nasal discharge, nonproductive cough, and sore throat. Persons with chronic rhinosinusitis complain of a constant pounding headache.

Which of the following disorders of the skin is most likely to respond to treatment with systemic antibiotics? a) Urticaria b) Verrucae c) Acne vulgaris d) Atopic dermatitis

Acne vulgaris The etiology of acne vulgaris is bacterial, and some clients may require treatment with systemic antibiotics. Urticaria (hives) and atopic dermatitis result from allergic and hypersensitivity processes. Verrucae, or warts, are of viral origin and thus unresponsive to antibiotic therapy.

Eczema is defined as which of the following? a) Thickened skin with exaggerated markings b) Skin with the surface scratched away c) Transient reddish skin plaque d) Acute, red, itchy, vesicular-crusted lesion

Acute, red, itchy, vesicular-crusted lesion Eczema manifests as inflammation and the residue left after vesicle rupture due to the dried residue of serum pus on the skin surface.

What is a primary intervention for the nurse to implement for a client with pelvic inflammatory disease ? a) Administer antibiotic therapy. b) Administer intravenous pain medication. c) Prepare the client for a hysterectomy. d) Prepare the client for laparoscopy.

Administer antibiotic therapy. While laparoscopy can provide definitive diagnosis, cases are usually diagnosed on symptomatology. Antibiotic therapy is usually sufficient for treatment. The client does not need a hysterectomy or IV pain medication.

Acute gastritis refers to a transient inflammation of the gastric mucosa that is most commonly associated with: a) Gastric reflux b) Alcohol intake c) Food allergies d) Diarrhea

Alcohol intake Acute gastritis refers to a transient inflammation of the gastric mucosa that is most commonly associated with local irritants such as bacterial endotoxins, alcohol, or aspirin. Gastritis associated with excessive alcohol consumption often causes transient gastric distress, which may lead to vomiting, bleeding, and hematemesis. Allergic response to ingested substances may cause acute itching, rash, vomiting, or diarrhea. Gastric reflux causes esophageal inflammation rather than gastritis.

Tissue destruction that is the result of the invasion of microorganisms is called: A. Resident flora intervention B. Pathogen destruction C. Receptor binding D. Infectious disease

D. Infectious disease

Which of the following individuals should be prioritized for receiving a seasonal influenza vaccination? a) An 86-year-old patient whose flu symptoms that have required hospitalization b) A 19-year-old man who was admitted to a hospital for an appendectomy c) An 81-year-old resident of a long-term care facility d) A neonate who was born in a busy, inner-city hospital in late October

An 81-year-old resident of a long-term care facility An older adult who lives in a nursing facility is a high-priority candidate for influenza vaccination. Infants younger than 6 months and persons who are acutely ill should not receive immunizations. Younger hospital clients may be vaccinated but are usually of lower priority than the elderly.

A patient is seen in the primary care office with eczema. Which is the best description to include about the skin assessment in the electronic medical record? a) Skin with the surface scratched away b) An acute, red, itchy, vesicular crusted lesion c) Thickened skin with exaggerated markings d) A transient reddish skin plaque

An acute, red, itchy, vesicular crusted lesion Atopic dermatitis (atopic eczema) is an itchy, inflammatory skin disorder that is characterized by poorly defined erythema with edema, vesicles, and weeping at the acute stage and skin thickening (lichenification) in the chronic stage.

A 12-year-old boy's severe wound that he received from a dog bite has begun to heal and currently shows no signs of infection. Which of the following processes occurred first during this process of repair by connective tissue deposition? a) Reorganization of fibrous tissue b) Angiogenesis c) Emigration of fibroblasts to the wound site d) Deposition of the extracellular matrix

Angiogenesis During the process of tissue repair by connective tissue deposition, angiogenesis precedes the emigration of fibroblasts, deposition of the extracellular matrix (ECM), and reorganization of the fibrous tissue.

A nurse is reviewing the admission assessment data of a client diagnosed with acute gastritis. The nurse determines that the condition most likely occurred as a result of: a) Arthritis treated with high levels of nonsteroidal anti-inflammatory (NSAIDs) agents b) A sinus infection that causes severe headaches c) Consuming a diet that is high in fiber and prepared with multiple spices d) Drinking a glass of red wine once a week

Arthritis treated with high levels of nonsteroidal anti-inflammatory (NSAIDs) agents Acute gastritis is most commonly associated with local irritants such as aspirin or other NSAIDs, alcohol, or bacterial toxins. A high-fiber diet and occasional alcohol consumption are not causes. The severe headache may cause stress or nausea but does not cause gastritis.

When teaching a patient with rheumatoid arthritis (RA), which of the following factors does the nurse explain is an underlying cause of this disease? a) Autologous antibodies b) Lysosomes c) Immunocompromised host d) Tissue necrosis

Autologous antibodies The pathogenesis of RA can be viewed as an aberrant immune response that leads to synovial inflammation and destruction of the joint architecture. Approximately 70-80% of people with the disease have a substance called rheumatoid factor, which is an autologous (self-produced) antibody that causes joint destruction.

Your neighbor is pregnant with her first child. She tells you that she needs to receive a RhIg injection. She understands why she must receive the injection but does not understand how the RhIg works. You tell her that the RhIg attaches to the: A. White blood cells B. Antigen in the mother's blood C. Antigen in the baby's blood D. Antibody in the mother's blood

B. Antigen in the mother's blood

Live microbes that have been modified with reduced virulence for incorporation into vaccines are referred to as being _____. A. Immune toxoids B. Attenuated C. Killed D. Compliment activated

B. Attenuated

Which of the following is not a mechanism by which pathogens cause disease? A. Direct destruction of the host cell B. Cellular restoration C. Interference with host cell's metabolic function D. Exposure to toxins

B. Cellular restoration

Tom goes to the clinic for sexually transmitted infection (STI) testing. He is told that he has chlamydia and gonorrhea. The health care practitioner indicates that this is a common ____. A. Chronic infection B. Coinfection C. Superinfection D. Recurrent infection

B. Coinfection

What is the difference between obligate and facultative parasites? A. Obligate can live independently of the host B. Facultative can live independently of the host C. Facultative are capable of producing exotoxins D. Obligate are much smaller in size than facultative

B. Facultative can live independently of the host

John's body has an exaggerated response to a bee sting. This response would be defined as: A. Autoimmunity B. Hypersensitivity C. Alloimmunity D. Failure of host defense mechanisms

B. Hypersensitivity

Alteration within the immune system can result from all of the following except: A. Failure of host defense mechanisms B. IgM immune memory C. Hypersensitivity D. Autoimmunity and alloimmunity

B. IgM immune memory

Which of the following is a major meningitis risk factor for college students? A. Homework B. Living in close quarters in dormatories or apartments C. Athletic competition D. Poor nutritional intake

B. Living in close quarters in dormatories or apartments

The redness, warmth, and swelling around the site of injury in Teri's arm are considered: A. Systemic signs of infection B. Local signs of infection C. Septicemia D. Chronic infection

B. Local signs of infection

What nutrition teaching would be included for the patient with viral hepatitis and impaired bile production? A. High protein diet B. Low fat diet C. Low carbohydrate diet D. High fiber diet

B. Low fat diet

Alexia has had a cold for the pat week. During school she coughs and sneezes. One of her friends, Donna, also becomes infected with the birus and develops symptoms 1 week later. When considering the chain of infection, the coughing and sneezing was most likely the: A. Vector B. Mode of transmission C. Reservoir D. Portal of entry

B. Mode of transmission

An 80 year old grandmother is admitted to the hospital for a hip fracture. During her stay on the orthopaedic unit, she develops an infection in her surgical wound. This type of infection that develops while a patient is in a hospital is called a(n): A. Chronic infection B. Nosocomial infection C. Ascending infection D. Recurrent infection

B. Nosocomial infection

Influenza is a virus that has the ability to change and adapt to its host. This ability to change genetic composition during replication in the human host cell is called: A. Host adaptation B. Reassortment C. Realignment D. Prodromal communication

B. Reassortment

Ellen, your neighbor, has contracted the influenza virus. She knows that you are taking pathophysiology and asks you how a simple virus can cause her sore throat, cough, and nasal congestion. You explain that the localized clinical manifestations that someone with influenza experiences are caused by: A. A septic response B. The inflammatory response and cell death C. The virus separation mode D. Nucleic acid metabolism

B. The inflammatory response and cell death

The early management recommendations for an asymptomatic person confirmed to have HIV antibodies would consist of which of the following? a) Quarantine to a special AIDS unit b) Baseline screening for viral load and CD4 counts c) Immediately starting antiviral medications d) Administration of intravenous anti-infective medications

Baseline screening for viral load and CD4 counts In an asymptomatic person, a baseline evaluation should be done to include an evaluation of viral load, CD4 counts, and physical examination. Antivirals would not be started until symptomatic and certain criteria met. AIDS patients are not quarantined.

Which of the following statements about opportunistic pathogen infection in AIDS is true? a) Most infections with opportunistic pathogens are not serious. b) Opportunistic pathogens often infect previously healthy people. c) HIV mainly attacks B cells. d) Becoming infected with an opportunistic pathogen is one requirement for the diagnosis of AIDS.

Becoming infected with an opportunistic pathogen is one requirement for the diagnosis of AIDS. Overt AIDS occurs when a person has a CDC4+ cell count of less than 200 cells per microliter of blood, or an AIDS-defining illness. Opportunistic infections involve common organisms that do not produce infection unless there is impaired immune function. Although a person with AIDS may live for many years after the first serious illness, as the immune system fails, these opportunistic illnesses become progressively more severe and difficult to treat.

Which of the following statements about opportunistic pathogen infection in AIDS is true? a) HIV mainly attacks B cells. b) Most infections with opportunistic pathogens are not serious. c) Becoming infected with an opportunistic pathogen is one requirement for the diagnosis of AIDS. d) Opportunistic pathogens often infect previously healthy people.

Becoming infected with an opportunistic pathogen is one requirement for the diagnosis of AIDS. The diagnosis of AIDS is confirmed when a patient's CD4+ count falls below 200 cells/μL or acquires an AIDS-defining illness (opportunistic disease). Opportunistic infections can infect healthy people, but they usually infect the immunocompromised, and they are serious. HIV attacks T cells.

A diagnosis of acquired immunodeficiency syndrome (AIDS) is identified when the CD4+ T cell count reaches which level? a) 200 to 499 cells/μL b) 800 to 1000 cells/μL c) 500 to 800 cells/μL d) Below 200 cells/μL

Below 200 cells/μL For a diagnosis of AIDS, the CD4+ cell count falls below 200 cells/μL. The other readings would not justify a finding of AIDS but can be found in HIV infection.

Major characteristics related to the adaptive immune system include all of the following except: A. Specificity and diversity B. Memory C. Antigen replacement D. Self ad non-self recognition

C. Antigen replacement

Amy is a 3 month old baby who has been hospitalized because of meningitis. Her treatment includes IV antibiotic therapy. After the therapy has started, her health status begins to decline quickly. You understand that the bacteria that causes meningitis is gram negative. Considering this, Amy's decline in health status is most likely due to the fact that: A. The gram negative bacteria react with gram positive bacteria B. The bacteria are not killed by the antibiotic C. As the bacteria were killed, endotoxins within the cell wall were released and they rtiggered a massive inflammatory response D. The bacteria are causing a hypersensitivity

C. As the bacteria were killed, endotoxins within the cell wall were released and they rtiggered a massive inflammatory response

Ty has leukemia. Because of the leukemia, his immune system suffers from an alteration called immunosuppression. Which of the following immunizations would carry the greatest risk for this type of patient? A. Immune serums B. Toxoids C. Attenuated vaccines D. Killed, inactivated vaccines

C. Attenuated vaccines

Which of the following is true about innate immunity? A. It is highly effective against many virus infections B. It is highly specific and adaptive C. It is the first to respond to pathogen challenge D. It is slower to respond than adaptive immunity

C. It is the first to respond to pathogen challenge

Caren is a 22 year old female. She come into the clinic because she suspects that she has a urinary tract infection. Most urinary tract infections are caused by the bacteria Escherichia coli. When E. Coli moves into another part of the body where it is not normally present and causes problems it is called a: A. Host cell B. Receptor C. Pathogen D. Parasite

C. Pathogen

Which pathogen is not known to elicit a notable immune response? A. Bacteria B. Viruses C. Prions D. Parasites

C. Prions

Besides the antibacterial medication that Teri was given, her health care provider also encouraged her to rest, take in plenty of oral fluids, and use acetaminophen for the pain. These types of treatments are called: A. Fungicidal treatment B. Prodromal treatment C. Symptomatic treatment D. Bactericidal treatment

C. Symptomatic treatment

Debbie develops a yeast infection. During the health history, she is asked if she has recently been on antibiotics. Debbie wonders what this has to do with yeast. The practitioner explains: A. Yeasts are destroyed by antibiotics B. Yeasts grow best when the patient is febrile C. Yeasts do not have to compete with resident bacteria and can overgrow D. The question was unrelated to her current concern

C. Yeasts do not have to compete with resident bacteria and can overgrow

The human immunodeficiency virus (HIV) acts on which of the following types of cells? a) White blood cells b) CD8+ T lymphocytes c) CD4+ T lymphocytes d) Red blood cells

CD4+ T lymphocytes HIV attacks the CD4+ T lymphocytes, the immune cells responsible for orchestrating and coordinating the immune response to infection. The other cells are not the target of the HIV virus

Which one of the following is an AIDS-defining condition? a) Any serious infection b) CD4+ count less than 200 cells/microliter of blood c) Malignancy of any organ d) Severe, prolonged diarrhea

CD4+ count less than 200 cells/microliter of blood Overt AIDS occurs when a person has a CD4+ cell count of less than 200 cells per microliter of blood or an AIDS-defining illness.

HIV virus preferentially infects which of the following cells? a) Platelets b) Red blood cells c) CD3+ lymphocytes d) CD4+T lymphocytes and nerve cells

CD4+T lymphocytes and nerve cells HIV infects a limited number of cell types in the body, including a subset of lymphocytes called CD4+T cells, macrophages, and dendritic cells.

A client has experienced a myocardial infarction with accompanying necrosis of cardiac muscle, a permanent tissue. What are the ramifications of the fact that cardiac muscle is a permanent tissue? a) Cardiac muscle cells will remain perpetually in the G1 stage of mitosis. b) Necrotic cells will be replaced with muscle cells that have limited metabolism. c) Regeneration of the patient's cardiac muscle will be exceptionally slow. d) Cells will not proliferate and will be replaced with scar tissue.

Cells will not proliferate and will be replaced with scar tissue. The cells in permanent tissues do not proliferate and, if destroyed, are replaced with fibrous scar tissue. The G0 stage of mitosis, not the G1 stage, involves quiescence.

Crohn's disease is recognized by sharply demarcated, granulomatous lesions that are surrounded by normal-appearing mucosal tissue. The nurse recognizes these lesions to be defined by which of the following descriptions? a) Mosaic b) Cobblestone c) Pyradimal d) Triangular

Cobblestone A characteristic feature of Crohn's disease is the sharply demarcated, granulomatous lesions that are surrounded by normal-appearing mucosal tissue. When there are multiple lesions, they are often referred to as skip lesions because they are interspersed between what appear to be normal segments of the bowel. The surface of the inflamed bowel usually has a characteristic "cobblestone" appearance resulting from the fissures and crevices that develop, surrounded by areas of submucosal edema.

A client underwent an open cholecystectomy 4 days ago, and her incision is now in the proliferative phase of healing. What is the dominant cellular process that characterizes this phase of the client's healing? a) Hemostasis and vasoconstriction b) Collagen secretion by fibroblasts c) Phagocytosis by neutrophils d) Keloid formation

Collagen secretion by fibroblasts The proliferative phase of wound healing is characterized by the action of fibroblasts. Hemostasis, vasoconstriction, and phagocytosis are characteristic of the inflammatory phase, whereas keloid formation is an abnormality in the remodeling phase.

Select the response that best describes a granulomatous inflammatory response to a foreign body: a) Normal inflammatory mechanisms to act on the foreign body. b) Foreign body to be digested and dissolved. c) Connective tissue encapsulates it and isolates it for removal. d) Macrophages attack the foreign object to dissolve it.

Connective tissue encapsulates it and isolates it for removal. Granulomatous inflammatory response to a splinter would involve the connective tissue encapsulating and isolating it for the removal. Each of the other options is part of the normal inflammatory response and would not apply in this situation.

A nurse reading a sigmoidoscopy report notes that a patient was found to have skip lesions. The nurse interprets this as an indication of which of the following? a) Zollinger-Ellison syndrome b) Ulcerative colitis c) Crohn's disease d) Peptic ulcer

Crohn's disease Skip lesions, demarcated granulomatous lesions that are surrounded by normal-appearing mucosal tissue, are a characteristic feature of Crohn's disease.

Why is Crohn's disease more likely to cause intestinal obstruction than ulcerative colitis?

Crohn's disease causes granulomas to form in the submucosal layer.

Rita is a 20 year old college student. She leads a very active lifestyle and belongs to the Hiking Club at school. Rita and a group of friends had planned an overnight hike in the desert. During their first day out, Rita was bitten by a snake. She was transported to the nearest hospital. As part of her treatment, Rita was given gamma globulin. Rita asks why they gave her this. You explain that the gamma globulin promotes: A. Acquired Active Immunity B. Naturally acquired passive immunity C. Artificially acquired active immunity D. Artificially acquired passive immunity

D. Artificially acquired passive immunity

John has developed an allergy to bee stings. Which immunoglobulin is activated during his body's response to a bee sting? A. IgG B. IgA C. IgM D. IgE

D. IgE

Teri's mom took her into the clinic to see their health care provider. The provider prescribed an antibacterial medication for Teri to take. Teri doesn;t understand how a pill that she takes is going to help the infection in her arm. Teri is told that an antibacterial medication works because it does all of the following except: A. Inhibits synthesis of the bacteria cell wall B. Damages the cytoplasmic membrane of the bacteria C. Disables nucleic acid metabolism or protein synthesis D. Promotes bacterial cell wall enlargement

D. Promotes bacterial cell wall enlargement

Which of the following helps to explain why certain populations become infected by certain pathogens and others do not? A. Average age of the population B. Viral reassortment C. Direct destruction D. Receptor binding

D. Receptor binding

The portal of entry into the human host for the influenza virus is the: A. Skin B. Mucous membrane C. Gastrointestinal tract D. Respiratory tract

D. Respiratory tract

Examples of a type II antibody mediated reaction include all of the following except: A. Blood transfusion reactions B. Graves' disease C. Hemolytic disease of the newborn D. Systemic lupus erythematosus

D. Systemic lupus erythematosus

The hospitalized burn patient wants to know why you need to remove his dressings every day. It is painful and he wants to avoid uncovering his burn injury. You explain that removing the dressings promotes-

Debridement

Depth of injury is important to determine with burns. You are in the sun too long without sunscreen and develop redness and blistering on your face, chest, and back. What depth of burn did you experience?

Deep partial-thickness burn

Which of the following clients, who have undergone a major surgery to repair a "leaking" colon with associated peritonitis, is most likely to experience enhanced wound healing as a result of his or her presurgery diet history? A client who: a) Eats a high-calorie diet and large amounts of red meat b) Is receiving total parenteral nutrition due to recurrent nausea c) Practices careful calorie control and who avoids animal fats d) Is a vegetarian and who eats organic foods when possible

Eats a high-calorie diet and large amounts of red meat Protein is important for many aspects of wound healing, including control of the inflammatory phase, fibroblast function, collagen and protein matrix synthesis, angiogenesis, and wound remodeling. A diet that is high in both calories and protein is known to enhance wound healing.

A patient is admitted with chronic gastritis. The nurse expects which of the following invasive tests to be performed to establish the presence of Helicobacter pylori (H. pylori)? a) Radiographic test b) Endoscopic biopsy c) Carbon urea breath test d) Stool antigen test

Endoscopic biopsy Methods for establishing the presence of H. pylori infection include the carbon urea breath test, the stool antigen test, and an endoscopic biopsy for urease testing. The invasive test is the endoscopic biopsy, which removes a tissue sample from the lining of the stomach.

Most common uncomplicated urinary tract infections are caused by ____ that enter through the urethra. a) Pseudomonas b) Group B Streptococcus c) Escherichia coli d) Staphylococcus aureus

Escherichia coli Most commonly urinary tract infections (UTIs) are caused by Escherichia coli bacteria that enter through the urethra. Other uropathic pathogens include Staphylococcus saprophyticus in uncomplicated UTIs, and both non-E. coli gram-negative rods (Proteus mirabilis, Klebsiella pneumoniae, Pseudomonas) and gram-positive cocci (Staphylococcus aureus, Group B Streptococcus) in complicated UTIs.

Which of the following phenomena best accounts for the increased presence of leukocytes at the site of inflammation? a) Increased numbers of leukocytes are released into circulation via cytokine stimulation. b) Existing leukocytes stick to the epithelial cells and move along blood vessel walls. c) Leukocytes are osmotically drawn from circulation into the interstitial space as a result of swelling. d) Epithelium expresses leukocyte stimulation factors in response to cell injury.

Existing leukocytes stick to the epithelial cells and move along blood vessel walls. During inflammation, leukocytes accumulate at the point of epithelial contact in the processes of margination, adhesion, and transmigration. This is not directly achieved by way of increased leukocyte production or release, nor by osmotic pressure. The epithelium does not produce leukocyte stimulation factors.

When caring for a client during the proliferative phase of wound healing, the nurse teaches the client that which of these processes is taking place? a) Scar tissue formation is complete. b) Fibroblasts secrete collagen for wound healing. c) White blood cells phagocytize bacteria and cellular debris. d) Fibrinogen is converted to fibrin to form a blood clot in the area.

Fibroblasts secrete collagen for wound healing. During the proliferative phase fibroblasts synthesize and secrete collagen and other intercellular elements needed to fill the wound space. Fibroblasts also produce a family of growth factors that induce angiogenesis and endothelial cell proliferation and migration.

A client asks the nurse why a scar developed after an injury. The best response would be: a) Stem cells b) Parenchymal tissue c) Cell apoptosis d) Fibrous tissue

Fibrous tissue Replacement of damaged tissue by connective (fibrous tissue) tissue leads to scar formation. Parenchymal tissue repair would not result in scar formation. Cell apoptosis refers to cell death, while stem cells are undifferentiated cells.

A patient comes in with a mild sunburn. Which best describes the sunburn? a) Second degree burn b) Photosensitivity reaction c) First degree burn d) Immune reaction to UV light

First degree burn First-degree burns (superficial partialthickness burns) involve only the outer layers of the epidermis. They are red or pink, dry, and painful. There usually is no blister formation. A mild sunburn is an example.

A nurse is teaching a client diagnosed with Crohn disease about potential complications. The most appropriate information for the nurse to include would be: a) Chronic constipation b) Fistula formation c) Excessive weight gain d) Difficulty swallowing

Fistula formation Complications of Crohn disease include fistula formation, abdominal abscess formation, and intestinal obstruction. Clients with Crohn disease are at risk for weight loss and/or diarrhea. The disease does not cause difficulty swallowing as it typically is in the bowel

Crohn's disease has a distinguishing pattern in the gastrointestinal (GI) tract. The surface has granulomatous lesions surrounded by normal-appearing mucosal tissue. A complication of the pattern includes which of the following? a) Constipation b) Dysphagia c) Fistula formation d) Rectal bleeding

Fistula formation In Crohn's disease all layers of the bowel are involved. Complications of Crohn's disease include fistula formation, abdominal abscess formation, and intestinal obstruction. Fistulas are tubelike passages that form connections between different sites in the GI tract.

A nursing student is cleaning and changing the dressing on a client's sacral ulcer. The student has vigorously cleansed the wound bed to remove all traces of the beefy, red tissue that existed in the wound bed. The student has most likely removed: a) The extracellular matrix b) Stem cells c) Granulation tissue d) Necrotic tissue

Granulation tissue Granulation tissue is a glistening red, moist connective tissue that fills the injured area while necrotic debris is removed. Stem cells will not exist in a wound bed and necrotic tissue would not be bright red. The student has likely done the client a disservice by cleansing aggressively.

A major difference between the acute and chronic inflammatory response is that in chronic inflammation-

Granulomas form around certain invaders

Crohn's type of inflammatory bowel disease is characterized by which of the following? a) Necrotic crypt abscesses b) Granulomatous lesions c) Ulcerative erosions d) Fibrotic smooth muscle

Granulomatous lesions Crohn's disease is a recurrent, granulomatous type of inflammatory response with formation of multiple sharply demarcated, granulomatous lesions that are surrounded by normal-appearing mucosal tissue. There is usually a relative sparing of the smooth muscle layers of the bowel, with marked submucosal layer inflammatory and fibrotic changes. Characteristic of ulcerative colitis (rather than Crohn's disease) are crypts of Lieberkuhn lesions in the base of the mucosal layer, formation of pinpoint mucosal hemorrhages, and development of crypt abscesses that become necrotic.

While teaching about HIV/AIDS to a group of high school seniors, the school health nurse will begin by explaining the basic facts which will likely include which of the following information? a) Like all viruses, HIV is a genetic material made from DNA with long molecules that carry genetic information. b) There are two types of HIV, but the one that is endemic to the U. S. is HIV type 2. c) HIV type 1 for some reason rarely develops into full-blown AIDS. d) HIV is different from other viruses since it is a retrovirus that selectively attacks the body's immune cells.

HIV is different from other viruses since it is a retrovirus that selectively attacks the body's immune cells. HIV is a retrovirus that selectively attacks the CD4+ T lymphocytes, the immune cells responsible for orchestrating and coordinating the immune response to infection. It must change from RNA to DNA through a series of stages in order to get in a cell and begin replication. HIV type 2 is endemic in West Africa but is rarely seen in other parts of the world. People with HIV-2 tend not to develop AIDS.

The nurse is caring for a client whose temperature is increasing. The nurse is aware that the client will also experience an increase in: a) Heart rate b) Respiratory rate c) White blood cell count d) Blood pressure

Heart rate Critical to the analysis of a fever pattern is the relation of heart rate to the level of temperature elevation. Normally, a 1°-C rise in temperature produces a 15-bpm (beats/minute) increase in heart rate (1°F, 10 bpm). The remaining options are not as closely related as is heart rate.

A 44-year-old man has been brought to the emergency department with severe electrical burns resulting from a workplace accident. The most immediate threat to this client's survival at this time is: a) Hemodynamic instability b) Infection c) Acute pain d) Decreased protein synthesis and impaired healing

Hemodynamic instability Although infection and the potential for sepsis are highly significant risks for clients with burns, the most immediate threat is hemodynamic instability. Pain control is essential, but inadequate pain control does not pose a direct threat to survival. The healing process is a later priority

A client tells the nurse that he is concerned about developing hepatitis after being exposed to contaminated feces, saliva, and food. The nurse is aware that the client is at risk for: a) Hepatitis D b) Hepatitis B c) Hepatitis C d) Hepatitis A

Hepatitis A Hepatitis A is normally transmitted through the fecal-oral route by drinking contaminated milk or water and eating shellfish from infected waters. Hepatitis B is transmitted through infected blood or serum, hepatitis C is transmitted by recreational injection drug use, and hepatitis D occurs largely to persons at high risk for HBV infection.

The nurse is caring for a client with hepatitis A who is tired of feeling ill. Which of these does the nurse convey to the client? a) Symptoms of hepatitis A last for 3-6 months. b) The client should consider accepting the hepatitis A vaccine. c) Without rest the client may progress to a chronic phase. d) Hepatitis A symptoms last for 8 weeks.

Hepatitis A symptoms last for 8 weeks. Hepatitis A is usually is a benign, self-limited disease, although it can cause acute fulminant hepatitis and death or need for transplantation in 0.15 to 0.2 percent of cases. Symptoms usually last approximately 2 months but can last longer.

A patient is diagnosed with Crohn's disease. The nurse instructs the patient on which of the following types of dietary needs? a) High-fat diet, low calorie b) High-residue diet c) High-calorie, vitamin, and protein diet d) High-fruit and vegetable intake

High-calorie, vitamin, and protein diet Nutritional deficiencies are common in Crohn's disease because of diarrhea, steatorrhea, and other malabsorption problems. A nutritious diet that is high in calories, vitamins, and proteins is recommended. Because fats often aggravate the diarrhea, it is recommended they be avoided. Elemental diets, which are nutritionally balanced but residue free and bulk free, may be given during the acute phase of the illness.

A deficiency in which of the following would result in an inhibition of the inflammatory response? a) Vitamin K b) Histamine c) B cells d) Helper T cells

Histamine Histamine is a key mediator in the inflammatory system, unlike helper T cells, B cells, or vitamin K.

A 24-year-old woman has gone to the OB-GYN clinic for her first visit since she found out she was pregnant. The clinician tested her blood type along with the usual prenatal testing. On a follow-up visit, the woman was told that she is Rh negative. When asked what that means for her baby, the nurse explains that Rh negative women lack RhD antigens on their erythrocytes but produce anti-D antibodies. As a result of this blood type, the a) If you and your baby have mismatched blood, it can invoke anaphylaxis in the baby. b) If the types are incompatible, severe antibody-mediated inflammation occurs. c) If the fetus is Rh-positive, maternal anti-D antibodies can coat fetal RBCs resulting in severe anemia. d) If blood types do not match, the baby's liver will produce extra cells to replace RBCs needed to oxygenate organs.

If the fetus is Rh-positive, maternal anti-D antibodies can coat fetal RBCs resulting in severe anemia. In utero, the development of erythroblastosis fetalis (Rh incompatibility) results when Rh negative women product anti-D antibodies. In Rh-positive fetus, maternal anti-D antibodies will coal fetal RBCs containing RhD, all them to be removed from fetal circulation by macrophage and monocytes mediated phagocytosis (destroying RBCs). RBCs are produced in bone marrow, not the liver, therefore answer C is incorrect.

A patient is being treated for an anaphylactic reaction after eating shrimp. Which of the following classes of antibodies mediates the anaphylaxis? a) IgA b) IgE c) IgG d) IgM

IgE Anaphylaxis is a catastrophic, systemic life-threatening IgE-mediated hypersensitivity reaction associated with the widespread release of histamine into the systemic circulation that produces massive vasodilation, hypotension, arterial hypoxia, and airway edema.

An elderly resident of an assisted-living facility has had his mobility and independence significantly impaired by the progression of his rheumatoid arthritis (RA). What is the primary pathophysiologic process that has contributed to this client's decline in health? a) Immunologically mediated joint inflammation b) Excessive collagen production and deposition c) A mismatch between bone resorption and remodeling d) Cytokine release following mechanical joint injury

Immunologically mediated joint inflammation The pathogenesis of RA can be viewed as an aberrant immune response that leads to synovial inflammation and destruction of the joint architecture. Paget disease is caused by abnormal bone resorption and remodeling, whereas collagen deposition underlies scleroderma. Osteoarthritis is believed to be initiated by mechanical injury and subsequent cytokine release.

The nurse is assessing a patient who has a unilateral obstruction of the urinary tract. The nurse anticipates the patient may develop: a) Increased urine output b) Increase in blood pressure c) Inability to control urination d) Excretion of dilute urine

Increase in blood pressure Hypertension is an occasional complication of urinary tract obstruction. It is more common in cases of unilateral obstruction in which renin secretion is enhanced, probably secondary to impaired renal blood flow. In these circumstances, removal of the obstruction often leads to a reduction in blood pressure. The urine output would be decreased and not diluted.

A client is experiencing the early stages of an inflammatory process and develops leukocytosis. The nurse recognizes this as an: a) Increase in cell production b) Increase in circulating neutrophils c) Decrease in blood supply to the affected area d) Decrease in eosinophils at the tissue injury site

Increase in circulating neutrophils Leukocytosis, or the increase in white blood cells, is a frequent sign of an inflammatory response, especially those caused by bacterial infection. Leukocytosis occurs due to an increase in circulating neutrophils and eosinophils. Leukocytosis does not occur because of increased cell production, and blood supply is typically increased as part of the inflammatory process.

You get a paper cut and experience pain at the site. This response is related to-

Increase perfusion at the site.

Dry, itchy plaques on her elbows and knees have prompted a 23-year-old woman to seek care. The clinician has subsequently diagnosed the client with psoriasis, a disorder that results from: a) Increased epidermal cell turnover b) An IgE-mediated immune reaction c) Hormonal influences on sebaceous gland activity d) Human papillomaviruses (HPV)

Increased epidermal cell turnover Psoriasis is characterized by increased epidermal cell turnover with marked epidermal thickening, a process called hyperkeratosis. Hives are caused by an IgE-mediated immune reaction, and HPV causes warts. Dysfunction of the sebaceous glands results in acne.

A nurse is assessing a patient's risk for developing a hospital acquired urniary tract infection (UTI) and determines that which of following places the patient at greatest risk? a) indwelling urinary catheter b) prostate enlargement c) increased sexual activity d) impaired bladder emptying

Indwelling urinary catheter Intrumentation and urinary catheterization are the most common predisposing factors for hospital acquired (nosocomial) urinary tract infections. The other items increase risk factors but are not necessarily related to hospitalization

Which of the following pathophysiologic processes occurs in cases of bacterial meningitis? a) Infection in the cerebrospinal fluid causes vasoconstriction and cerebral hypoxia. b) Inflammaion allows pathogens to cross into the cerebrospinal fluid. c) Trauma introduces skin-borne pathogens to the cerebrospinal fluid. d) Infection in the cerebrospinal fluid causes spinal cord compression and neurologic deficits.

Inflammaion allows pathogens to cross into the cerebrospinal fluid. In the pathophysiologic process of bacterial meningitis, the bacterial organisms replicate and undergo lysis in the CSF, releasing endotoxins or cell wall fragments. These substances initiate the release of inflammatory mediators, which set off a complex sequence of events permitting pathogens, neutrophils, and albumin to move across the capillary wall into the CSF. Cerebral hypoxia does not result directly from meningitis, and the causative pathogens are not introduced from the skin nor is trauma an initiating event. Spinal cord compression is not an expected consequence of meningitis.

A patient is admitted to the medical surgical unit with a history of inflammatory bowel disease. The nurse knows that the clinical manifestations of both Crohn's disease and ulcerative colitis are the result of activation of which of the following cells? a) Alpha b) Inflammatory c) Parietal d) Beta

Inflammatory The term "inflammatory bowel disease" is used to designate two related inflammatory intestinal disorders: Crohn's disease and ulcerative colitis. Both diseases produce inflammation of the bowel. The clinical manifestations of both Crohn's disease and ulcerative colitis are ultimately the result of inflammatory cells with elaboration of inflammatory mediators that cause nonspecific tissue damage.

Crohn's disease is treated by several measures. Treatment with corticosteroids focuses on which of the following? a) Inflammatory suppression b) Immune suppression c) Decreased bleeding tendency d) Increased appetite

Inflammatory suppression Treatment methods focus on terminating the inflammatory response and promoting healing, maintaining adequate nutrition, and preventing and treating complications. Several medications have been successful in suppressing the inflammatory reaction, including corticosteroids, sulfasalazine, metronidazole, azathioprine, 6-mercaptopurine, methotrexate, and infliximab.

The nurse is assessing a client who states he felt fine all day and then within five minutes, began to feel extremely weak and tired and began having a fever. What do these symptoms indicate that the client may be experiencing? a) Viral pneumonia b) Influenza c) Common cold d) Reye syndrome

Influenza One distinguishing feature of an influenza viral infection is the rapid onset, sometimes in as little as 1 to 2 minutes, of profound malaise. Viral pneumonia is a complication of influenza. Reye syndrome is a complication of influenza that is extremely rare and is found mostly in children after being give aspirin as an antipyretic. The common cold has more of a slow onset.

A client asks the nurse if it is possible to contract influenza by being exposed to wound secretions. On what knowledge should the nurse base her response to the client? a) Influenza transmission occurs by inhalation of droplet nuclei. b) Influenza may be contracted by touching a surface that an infected person has touched. c) Influenza transmission occurs through contact with the skin of an infected person. d) Influenza is transmitted by exposure to urine, blood, and body secretions.

Influenza transmission occurs by inhalation of droplet nuclei. As with many viral respiratory tract infections, influenza is more contagious than bacterial respiratory tract infections. In contrast to the rhinoviruses, transmission occurs by inhalation of droplet nuclei rather than touching contaminated objects.

Which of the following is the most common cause of acute gastritis?

Ingestion of aspirin, alcohol, or other chemicals

A school age child with a history of asthma has brought a note home from school stating there has been one case of meningitis (Neisseria meningitides) in the school. Since the mother is a nurse, she is very concerned since she knows the portal of entry of this pathogen is: a) Direct contact with a contaminated object such as a pencil. b) Inhalation via the respiratory tract such as through breathing or yawning. c) Through a cut or abrasion that may occur on the playground. d) Ingestion such as when children share their drink with their friends.

Inhalation via the respiratory tract such as through breathing or yawning. Neisseria meningitides is one of a number of pathogens that invade the body through the respiratory track.

A 44 year-old male hospital client with a diagnosis of end-stage acquired immunodeficiency syndrome (AIDS) has been placed on neutropenic precautions that limit his interaction with visitors, staff and other clients. What is the underlying rationale for these precautions? a) Cyclic neutropenia limits his body's ability to fight various infections. b) Neutropenia limits the ability of his CD4 helper cells to present antigens. c) Insufficient levels of neutrophils make him particularly susceptible to infections. d) His antibody-mediated immunity is compromised by his low production of neutrophils.

Insufficient levels of neutrophils make him particularly susceptible to infections. A decrease in the number of neutrophils places an individual at risk for infection. Neutrophils are not directly involved in the antibody-mediated immune process and his neutropenia is infection-related, not cyclic.

HIV is considered to be a retrovirus because: a) it reproduces at a rapid rate. b) it carries its genetic information in ribonucleic acid (RNA) rather than deoxyribonucleic acid (DNA). c) it converts to a primitive form of virus when duplicated. d) it carries a genetic marker for a previously discovered virus that was a source of an epidemic in an earlier time period.

It carries its genetic information in ribonucleic acid (RNA) rather than deoxyribonucleic acid (DNA). Like other retroviruses, HIV carries its genetic information in ribonucleic acid (RNA) rather than deoxyribonucleic acid (DNA). In the process of taking over the CD4+ T-cell, the virus attaches to receptors on the CD4+ cell, fuses to and enters the cell, incorporates its RNA into the cell's DNA, and then uses the CD4+ cell's DNA to reproduce large amounts of HIV, which are released into the blood.

What is the purpose of a rise in body temperature? a) It inhibits the growth of many pathogenic microbial agents. b) It decreases blood loss. c) It increases the risk of infection. d) It increases proliferation of viruses

It inhibits the growth of many pathogenic microbial agents. There is little research to support the belief that fever is harmful unless the temperature rises to extreme levels. It has been shown that small elevations in temperature, such as those that occur with fever, enhance immune function. Many of the microbial agents, including those that cause infection, grow best at normal body temperatures, and their growth is inhibited by temperatures in the fever range.

Which of the following signs and symptoms should prompt a 29-year-old woman's primary care provider to assess for systemic lupus erythematosus (SLE)? a) Chronic nausea and vomiting that is unresponsive to antiemetics b) A history of thromboembolic events and varicose veins c) Dysmenorrhea and recent spontaneous abortion d) Joint pain and increased creatinine and blood urea nitrogen

Joint pain and increased creatinine and blood urea nitrogen Renal involvement occurs in approximately one half to two thirds of persons with SLE, and arthralgia is a common early symptom of the disease. Although the manifestations of SLE are diffuse, these do not typically include alterations in hemostasis, gastrointestinal symptoms, dysmenorrhea, or miscarriage.

A 36-year-old male who is positive for HIV antibodies notices purplish spots on his upper body. Which of the following terms is used to identify these areas as an opportunistic infection? a) Kaposi's sarcoma b) Multifocal leukoencephalopathy c) Liver spots d) Toxoplasmosis

Kaposi's sarcoma Purplish spots identified in a HIV-positive person on the trunk, neck, or head are uniquely Kaposi's sarcoma, an opportunistic cancer that is 2000 times more likely to occur in immunocompromised persons. Toxoplasmosis and multifocal leukoencephalopathy are opportunistic infections but are not skin lesions. Liver spots occur with aging and sun exposure and are not solely related to those with HIV.

The nurse is caring for a client with an accumulation of 2.5 cm. of darkened tissue scar over the area of a 3 mm. injury. How does the nurse correctly document this finding in the medical record? a) Tumor b) Hernia c) Keloid d) Thrombus

Keloid Keloids are tumor-like masses caused by excess production of scar tissue. The tendency toward development of keloids is more common in African Americans and seems to have a genetic basis.

While the nurse is performing a skin assessment on an African American client, the nurse notes that the client has a healed wound on the leg but that the wound has an excess of scar tissue. The nurse documents this as which of the following? a) Remodeling b) Proud flesh c) Epithelialization d) Keloid

Keloid The nurse doucments the existence of excess of scar tissue on a healed wound as a Keloid. These are more common in African Americans. Proud flesh is an excess of granulation tissue in a healing wound. Remodeling is the third phase of wound healing. Epithelialization is the migtation, proliferation, and diffentiation of epeithilial cess on th wound edges and occurs during the proliferative phase of wound healing.

5. Which is not a local manifestation of acute inflammation?

Leukocytosis

The emergency room doctor suspects a client may have bacterial meningitis. The most important diagnostic test to perform would be: a) Sputum culture b) Blood cultures c) CT of the head d) Lumbar puncture

Lumbar puncture The diagnosis of bacterial meningitis is confirmed with abnormal CSF findings. Lumbar puncture findings, which are necessary for accurate diagnosis, include a cloudy and purulent CSF under increased pressure. The other options do not confirm the diagnosis.

In contrast to acute inflammation, chronic inflammation is characterized by which of the following phenomena? a) Profuse fibrinous exudation b) Metabolic and respiratory alkalosis c) A "shift to the left" of granulocytes d) Lymphocytosis and activated macrophages

Lymphocytosis and activated macrophages Chronic inflammation requires lymphocytes and macrophages to remain in large numbers for the high use of immune cells. Chronic inflammation is associated with fibroblast proliferation instead of exudations. A "shift to the left" is characteristic of acute inflammation with a high neutrophil count. Inflammation, with continued cell injury, is a source of metabolic and respiratory (if the lungs are the site of inflammation) acidosis.

Systemic lupus erythematosus is best characterized by which one of the following? a) The main immune problem is anaphylactic (type I immune) reaction. b) It is principally a disease of men. c) Most patients have antinuclear antibodies present in their blood. d) It is a consequence of streptococcal infection.

Most patients have antinuclear antibodies present in their blood. Patients diagnosed with systemic lupus erythematosus will most commonly have antinuclear antibodies in their blood. Lupus is primarily a disease of women and it is an autoimmune reaction.

A patient presents with joint pain and suspected rheumatoid arthritis (RA). Which of the following is the appropriate nursing action? a) Increase in the client's activity to keep joints mobile b) Muscle-strengthening exercises to support joints c) Admission to the intensive care unit for monitoring d) Intravenous narcotics

Muscle-strengthening exercises to support joints Patients with RA can focus on muscle strengthening to support joints. Range-of-motion exercises and isometric exercises may be used to strengthen muscles. Rest, rather than increased activity, is beneficial. IV narcotics are not generally used, and admission to the intensive care unit is not needed. Heat and cold modalities can be used as well as teaching the patient about proper body mechanics.

During a lecture on inflammation, the physiology instructor discusses the major cellular components involved in the inflammation response. The instructor asks, "Which of the following cells arrives early in great numbers?" The student with the correct response is: a) Neutrophils b) Basophils c) Lymphocytes d) Monocytes

Neutrophils Neutrophils are the primary early arrival cells and are signified by an elevated neutrophil count that includes mature (PMNs) and immature (bands) cell forms. Basophils respond later. Lymphocytes have a slower arrival and stay longer. The half-life of circulating monocytes is about a day, after which they begin to migrate to the site of injury and mature into larger macrophages, which have a longer half-life and greater phagocytic ability than do blood monocytes.

A patient presented to the emergency department of the hospital with a swollen, reddened, painful leg wound and has been diagnosed with methicillin-resistant Staphylococcus aureus (MRSA) cellulitis. The patient's physician has ordered a complete blood count and white cell differential. Which of the following blood components would the physician most likely anticipate to be elevated? a) Neutrophils b) Eosinophils c) Basophils d) Platelets

Neutrophils Increased neutrophils are associated with inflammation in general and bacterial infections in particular. Platelets play a role in inflammation but their levels would not rise to the same extent as would neutrophils. Eosinophils are not strongly associated with bacterial infection and basophils would not increase to the same degree as neutrophils.

A school nurse is teaching a class on immunity. Which statement contains an accurate explanation about cellular defenses? a) Active transport draws invading organisms through cell channels to be destroyed. b) Neutrophils engulf invading organisms where lysosomes break them down. c) Ion channels allow rapid polarity changes, which inactivate invading organisms. d) Cells release enzymes into the extracellular fluid that degrade invading organisms.

Neutrophils engulf invading organisms where lysosomes break them down. Phagocytosis is when a cell, such as macrophages or neutrophils, engulfs an invading organism or damaged cells. Once inside the cell, the encapsulated particle is broken down by lysosomal enzymes.

The nurse is caring for a patient with newly diagnosed systemic lupus erythematosus (SLE). Which of the following over-the-counter medications does the nurse recognize is useful in treating inflammation, arthritis, and pleuritis? a) Hydroxychlorquine b) Belimumab c) Cycloposphamide d) Nonsteroidal anti-inflammatory drugs (NSAIDs)

Nonsteroidal anti-inflammatory drugs (NSAIDs) Treatment with medications may be as simple as a drug to reduce inflammation, such as an NSAID. NSAIDs can control fever, arthritis, and mild pleuritis.

A female patient is being seen in the ED with complaints of lower abdominal pain, dyspareunia, back pain, purulent cervical discharge, and the presence of adnexal tenderness and painful cervix on bimanual pelvic examination. The nurse suspects the patient has pelvic inflammatory disease (PID), and will assess the patient for which of the following? a) Number of sexual partners b) Use of oral contraceptives c) Dietary habits d) Toileting habits

Number of sexual partners PID is a polymicrobial infection of the upper reproductive tract (uterus, fallopian tubes, or ovaries) associated with the sexually transmitted organisms such as N. gonorrhoeae or C. trachomatis as well as endogenous organisms, including anaerobes, such as Haemophilus influenza, enteric gram-negative rods, and streptococci. Factors that predispose women to the development of PID include an age of 16 to 24 years, nulliparity, history of multiple sexual partners, and previous history of PID. The symptoms of PID include lower abdominal pain, which may start just after a menstrual period; dyspareunia; back pain; purulent cervical discharge; and the presence of adnexal tenderness and exquisitely painful cervix on bimanual pelvic examination.

The nurse is caring for a client who has AIDS and suffers from wasting syndrome. Which of the following is the priority nursing consideration for this client? a) Nutrition b) Oxygenation c) Pain d) Skin care

Nutrition The wasting syndrome is an AIDS-defining illness and is common in people with HIV infection or AIDS. Wasting syndrome is characterized by involuntary weight loss of at least 10 percent of baseline. Treatment for wasting includes nutritional interventions.

What is the one definitive test to diagnoses rheumatoid arthritis?

One test is not definitive

Tissue repair through regeneration occurs in which cells? a) Stromal b) Parent c) Parenchymal d) Daughter

Parenchymal Tissue repair can take the form of regeneration in which injured parenchymal cells are replaced with cells of the same type, sometimes leaving no residual trace of previous injury. The stromal cells are found in the supporting connective tissues, blood vessels, fibroblasts, nerve fibers, and the extracellular matrix. These cell populations rely on progenitor or parent cells of the same lineage that have not yet differentiated. Progenitor cells are sufficiently differentiated so that their daughter cells are limited to the same cell line, but they have not reached the point of differentiation that precludes the potential for active proliferation.

The nurse is evaluating her patient's risk for an urinary tract infection. Which of the following patients has the highest risk? a) Patient with septicemia b) Patient with infective endocarditis c) Patient with high levels of glucose in the urine d) Patient with obstructed urinary outflow from the kidney

Patient with obstructed urinary outflow from the kidney Factors that contribute to the development of ascending infections of the urinary tract are outflow obstruction, catheterization and urinary instrumentation, vesicoureteral reflux, pregnancy, and neurogenic bladder.

A 33-year-old patient who is a long-term intravenous user of heroin has been recently diagnosed with hepatitis C. Which of the following portals of entry most likely led to the patient's infection? a) Penetration b) Direct contact c) Vertical transmission d) Ingestion

Penetration Direct inoculation from intravenous drug use, and the accompanying disruption in the integrity of the body's surface barrier, is an example of entry by penetration. Vertical transmission occurs from mother to child, and neither direct contact nor ingestion is a likely source of this patient's infection.

Which of the following best describes the consumption of a foreign substance, such as bacteria, in a cell? a) Meiosis b) Anaerobic metabolism c) Hemolysis d) Phagocytosis

Phagocytosis Phagocytosis refers to the consumption of a foreign substance.. Energy transformation takes place without oxygen is referred to as anaerobic metabolism. Meiosis a type of cell division that results in two daughter cells each with half the chromosome number of the parent cell. Hemolysis refers to the destruction of red blood cells which leads to the release of hemoglobin from within the red blood cells into the blood plasma.

A patient is being seen in the dermatologist clinic for psoriasis. What term should the nurse select on the pull down menu in the electronic medical record to describe the patient's lesions associated with psoriasis? a) Cysts b) Nodules c) Pustules d) Plaques

Plaques Psoriasis has raised, well-demarcated, erythematous plaques with adherent silvery scales. Disorders of the pilosebaceous unit occur most frequently in adolescents and manifests as nodules, pustules or cysts.

Which one of the following is a common opportunistic infection in AIDS patients? a) Myalgia b) Chancroid c) Syphilis d) Pneumocystis jiroveci pneumonia (PCP)

Pneumocystis jiroveci pneumonia (PCP) PCP is a very common opportunistic infection in individuals with AIDS. It is a common bacterial found in households and does not routinely effect someone with a healthy immune system. Chancroid and syphilis are infections that do not occur commonly in the population in general including those with AIDS. Myalgia is not an infection, but it is muscle pain, a common symptom of an infection.

Which of the following is the main effect of HIV infection? a) Poor natural killer cell function b) Poor helper T-cell function c) Poor suppressor T-cell function d) Poor B-cell function

Poor helper T-cell function Helper T cells are also known as CD4+ T cells. These CD4+ T cells are necessary for normal immune function and are the main target of HIV.

Inflammation is ultimately needed to-

Prepare the site for healing

When an Rh-negative mother has been sensitized and is pregnant with an Rh-positive fetus, what happens to the fetus? a) Nothing, this is normal b) Plasma volume depletion c) Bilirubin deficiency d) Profound red cell hemolysis

Profound red cell hemolysis This situation is totally detrimental to the health of the fetus. Rh incompatibility and production of antibodies by the mother will result in life-threatening fetal red cell hemolysis. The bilirubin will be sufficiently elevated to cause brain damage. The infant will have massive edema from a lack of albumin production

A client sustained an injury 3 days ago. The nurse is assessing the status of the wound. The nurse anticipates the phase of healing to be: a) Collagen b) Remodeling c) Inflammatory d) Proliferative

Proliferative The proliferative phase begins within 2 to 3 days of an injury and focuses on building new tissue to fill the wound. The inflammatory phase begins at the time of the injury occurrence. The remodeling phase occurs approximately 3 weeks after the injury, and there is not a collagen phase.

The nurse is assessing a 9 day old wound. The nurse notes that the edema surrounding the wound has diminished and that the wound is now blanchable. The nurse doucments that the wound is in which stage of the healing process? a) Remodeling b) Inflammatory c) Proliferative d) Secondary

Proliferative During the proliferative phase there is continued accumulation of collagen proliferation of fibroblast,diminished edema and the wound begins to blanch as the small blood vessels become thrombosed and degenerate.

A nurse who is caring for a patient with end-stage hepatitis is aware that this patient is at risk for prolonged or excessive bleeding due to a decrease in which of the following? a) Adenosine triphosphate b) Von Willebrand factor c) Adenosine diphosphate d) Prothrombin

Prothrombin Hepatitis directly affects the function of the liver, which is responsible for the synthesis of prothrombin, an essential component in both the intrinsic and extrinsic coagulation pathways. Von Willebrand factor is synthesized by the endothelium; adenosine triphosphate and adenosine diphosphate are produced by the platelets.

A patient has recently been diagnosed with H. pylori gastritis. The nurse knows that this form of gastritis is usually treated with a combination of an antibiotic and: a) Calcium carbonate, an antacid. b) Lactulose, to reduce the blood ammonia levels. c) Anti-anxiety medications. d) Proton pump inhibitors.

Proton pump inhibitors. H. pylori is associated with an increased risk of gastric adenocarcinoma, gastric atrophy and peptic ulcer. It is less likely to contribute to IBD, esophagitis or diverticular disease. Eradication of H. pylori is difficult. Treatment required a combination therapy that includes the use of antibiotics and a proton pump inhibitor. The proton pump inhibitors have direct antimicrobial properties against H. pylori. Anti-anxiety medications will not kill the bacteria. H. pylori is not associated with elevated blood ammonia levels. Calcium carbonate is usually given to relieve heartburn caused by GERD.

Unlike the Crohn's type of inflammatory bowel disease, the ulcerative colitis type is characterized by which of the following? a) Skip lesions b) Gastric ulcers c) Pseudopolyps d) Steatorrhea

Pseudopolyps Unlike Crohn's disease, which can affect various sites in the gastrointestinal tract, ulcerative colitis is confined to the rectum and colon. As a result of the inflammatory process, the mucosal layer often develops tonguelike projections that resemble polyps and are therefore called pseudopolyps. Nutritional deficiencies are common in Crohn's disease because of diarrhea, steatorrhea (fatty stools), and other malabsorption problems. Crohn's disease causes granulomatous changes, often referred to as skip lesions because they are interspersed between what appear to be normal segments of the bowel.

A nurse is caring for a patient with rheumatoid arthritis (RA) who states she "caught" this from her mother. Which of the following statements is the correct explanation of how this type of arthritis develops? a) RA is related to overuse of a joint. b) RA has an autoimmune and genetic predisposition. c) RA is a disease of the elderly, especially women. d) RA is caused by a virus, transmitted in utero.

RA has an autoimmune and genetic predisposition. The importance of genetic factors in the pathogenesis of RA is supported by the increased frequency of the disease among first-degree relatives. Evidence suggests that this genetic predisposition causes joint inflammation. (less)

The presence of granulation tissue at a wound site is identified by: a) Purulent drainage b) Fibrous scar tissue c) Red, moist tissue d) White wound bed

Red, moist tissue Granulation tissue is a glistening red, moist connective tissue that fills the injured area. The other options reflect the presence of scarring, infection, or necrosis.

A client in the acute stage of inflammation will experience vasodilation of the arterioles and congestion in the capillary beds. The nurse would assess the client's skin for: a) Decreased sensation at the site b) Coolness c) Redness d) Increased bacterial infection

Redness Vasodilation of the arterioles and congestion of the capillary beds result in an increased pooling of blood leading to redness. The site would also have increased painful sensation and be warmer to touch. It would not result in an increase in bacterial load

A diabetic client has injured his foot while walking barefoot on the lawn. On admission, which of the following assessment findings would be considered a localized cardinal sign of acute inflammation? a) Temperature of 101°F b) Urine output of less than 500mL/24 hours (low) c) Fatigue with listlessness d) Redness and edema at the injured site

Redness and edema at the injured site Localized manifestations include redness, swelling, and heat. Fever and fatigue are systemic manifestations of acute inflammation. Low urine output is not a localized sign but could be a systemic manifestation if the client goes into septic shock.

The nurse is caring for a client who is receiving exogenous corticosteroids for rheumatoid arthritis. Recognizing that hormone levels are regulated by negative feedback, which of these laboratory test results does the nurse anticipate uncovering when reviewing the medical record? a) Increase in TSH b) Increase in ACTH c) Reduction in TSH d) Reduction in ACTH

Reduction in ACTH The function of the negative feedback system is similar to that of the thermostat in a heating system. When the sensors detect a decrease in hormone levels, they initiate changes that cause an increase in hormone production. When hormone levels rise above the set point of the system, the sensors cause hormone production and release to decrease, driving the stimulating hormone down.

The objective of tissue regeneration in wound healing is to: a) Remodel the area to preinjury appearance b) Fill in the gap caused by tissue damage c) Minimize pain resulting from the injury d) Restore injured tissue to its original structure

Restore injured tissue to its original structure While the objective of the healing process is to fill the gap created by tissue destruction and to restore the structural continuity of the injured part, tissue regeneration refers the restoration of injured tissue to its normal structure and function by proliferation of adjacent surviving cells. Pain and appearance are not addressed by regeneration.

If an Rh-negative mother is giving birth to an Rh-positive infant, the nurse should be prepared to administer: a) Antihistamines like Benadryl. b) Alpha interferon. c) A monoclonal antibody like infliximab. d) Rh immune globulin.

Rh immune globulin. The injection of Rh immune globulin prevents sensitization in Rh-negative mothers who have given birth to Rh-positive infants if administered at 28 weeks' gestation and within 72 hours of delivery, abortion, or genetic amniocentesis. Antihistamines, alpha interferon, or infliximab are not used in this situation.

A nurse is assessing a client for the classic signs of acute inflammation. The nurse would assess the client for: a) Rubor, swelling, and pain b) Pain, pulselessness, and edema c) Cyanosis, heat, and swelling d) Parasthesis, redness, and coolness

Rubor, swelling, and pain The classic signs of inflammation are rubor (redness), tumor (swelling), calor (heat), and dolor (pain). The remaining options are more characteristic of symptomatology resulting from circulatory dysfunction.

A client visits the OB/GYN clinic complaining of low abdominal pain, purulent cervical discharge, and painful intercourse. The health care worker (HCW) diagnoses pelvic inflammatory disease. The HCW educates the client about that fact that this disease may be associated with: a) Chronic endometriosis b) Serous luteal ovarian cysts c) Ruptured tubal pregnancy d) STI polymicrobial infection

STI polymicrobial infection Pelvic inflammatory disease (PID) is an inflammation and polymicrobial infection of the upper reproductive tract that involves the uterus (endometritis), fallopian tubes (salpingitis), or ovaries (oophoritis) associated with sexually transmitted and endogenous organisms. Endometriosis is the condition in which functional endometrial tissue is found in ectopic sites outside the uterus; the displaced endometrial tissue may cause localized inflammation. Ruptured tubal pregnancy causes salpingitis and inflammation unrelated to sexually transmitted disease (STD) infections. Benign ovarian epithelial tumors are almost always serous or mucinous. Serous (fluid-filled) luteal or follicular ovarian cysts are noninfective, are noninflammatory, and frequently resolve spontaneously without treatment.

A 14-year-old presents to the dermatologist clinic with a severe case of acne. The nurse explains that this acne is a result of inflamed a) Apocrine glands b) Sebaceous glands c) Follicular bulbs d) Dermal papillae

Sebaceous glands The sebaceous glands are the structures that become inflamed in acne. Growth of the hair is centered in the bulb (i.e., the base) of the hair follicle, which is just one part of the hair structure. Apocrine sweat glands are located deep in the dermal layer and open through a hair follicle. Dermal papillae minimize separation of the dermis and the epidermis and contain capillary venules that nourish the epidermal layers of the skin.

A wound is 6cm x 6cm x 4cm. A wound with these dimensions needs to heal through-

Secondary intention

A client has a watery fluid leaking from a site of inflammation. The nurse would document this type of exudate as: a) Fibrinous b) Hemorrhagic c) Suppurative d) Serous

Serous Serous exudate is a watery fluid low in protein content that results from plasma entering the inflammatory site. Hemorrhagic exudate is red or blood tinged related to damage to blood vessels. Suppurative exudate is composed of degraded white blood cells and tissue debris, leaving the fluid pus-like. Fibrinous exudate is thick and sticky meshwork fluid.

Inflammatory bowel disease (IBD) is used to designate two related inflammatory intestinal disorders: Crohn's disease and ulcerative colitis. The nurse recognizes the difference between the distribution pattern between Crohn's disease and ulcerative colitis. Which of the following patterns describes Crohn's disease? a) Development of cancer b) Continuous involvement of the colon starting at the rectum c) Primarily rectum and colon involvement d) Skip lesions

Skip lesions Distribution patterns of disease with skip lesions in Crohn's disease, and continuous involvement of the colon starting at the rectum in ulcerative colitis. Crohn's disease primarily affects the ileum and secondarily the colon, and the development of cancer is uncommon. Ulcerative colitis primarily affects the rectum and left colon, and development of cancer is relatively common.

In explaining a papule to a patient, a nurse defines it as being which of the following? a) Small, raised solid mass with a circumscribed border b) Closed, rounded space containing fluid c) Flat-topped, solid lesion d) Small abscess

Small, raised solid mass with a circumscribed border A papule is an elevated, palpable, solid mass with a circumscribed border less than 0.5 cm.

Select the option that best describes the type of tissue that is capable of regeneration when appropriately stimulated. a) Continuously dividing b) Permanent c) Labile d) Stable

Stable Stable tissues contain cells that normally stop dividing when growth ceases. However, these cells are capable of undergoing regeneration when confronted with an appropriate stimulus and are thus capable of reconstituting the tissue of origin. Continuously dividing or labile tissues are those in which the cells continue to divide and replicate throughout life, replacing cells that are continually being destroyed. The cells in permanent tissues do not proliferate. The cells in these tissues are considered to be terminally differentiated and do not undergo mitotic division in postnatal life.

When caring for a client with a wound that is healing by primary intention the nurse recognizes which of these best describes this type of wound? a) Fibrin clot (scab) b) Surgical incision c) Abcess d) Burn injury

Surgical incision A sutured surgical incision is an example of healing by primary intention. Larger wounds (e.g., burns and large surface wounds) or wounds purposely left open due to infection heal by secondary intention.

A nurse is caring for a patient admitted with a malar rash on the nose and cheeks. The nurse recognizes that this rash is characteristic of which of the following disease processes? a) Rheumatoid arthritis b) Systemic lupus erythematosus c) Scleroderma d) Sarcoidosis

Systemic lupus erythematosus The acute skin lesions include the classic malar or "butterfly" rash on the nose and cheeks.

Which of the following is the most important discharge instruction for a female patient with pelvic inflammatory disease (PID)? a) Drink plenty of fluids b) Get adequate rest c) Eat a balanced diet d) Take all of the antibiotic

Take all of the antibiotic PID is an infection of the upper reproductive tract associated with sexually transmitted organisms. The most important intervention to minimize complications is to complete the course of antibiotic therapy. Rest, fluids, and a well-balanced diet all enhance recovery.

Select the statement that best describes systemic lupus erythematosus (SLE). SLE is characterized by which of the following? a) Exposure to UV light will aid in decreasing symptoms. b) T-cell-mediated response to an immunologic trigger c) The formation of autoantibodies and immune complexes (type III hypersensitivity) d) An onset that is always sudden and presence of symptoms are constant and consistent.

The formation of autoantibodies and immune complexes (type III hypersensitivity) SLE is characterized by the formation of autoantibodies and immune complexes (type III hypersensitivity; T-cell-mediated response to an immunologic trigger occurs in RA). SLE may be acute or insidious, and the course of the disease is characterized by exacerbations and remissions. Exposure to UV light, specifically UVB associated with exposure to the sun or unshielded fluorescent bulbs, may trigger exacerbations.

A client has been diagnosed with a gram-negative bacillus in his blood cultures. The health care providers know these bacteria may produce clinical manifestations such as high temperature, high respiratory rate, and low blood pressure. These manifestations are primarily caused by: a) The bacteria causing a decrease in protein synthesis and function b) Disrupting the sodium/potassium ATPase pump c) The outer layer of the bacterial membrane acting as an endotoxin d) Interrupting oxidative metabolism processes

The outer layer of the bacterial membrane acting as an endotoxin Bacteria and viruses can replicate within a cell, thus perpetuating the injuries. Gram-negative bacilli have unique characteristics in the structure of the outer membrane. The outer leaflet of the membrane has a lipid portion that acts as an endotoxin. If this bacillus enters the circulatory system, it causes a toxic reaction, with the sufferer developing a high temperature, high respiration rate, and low blood pressure. Other agents that are injurious to cells are unable to replicate in the cell, but they may disrupt the sodium/potassium pump, interrupt oxidative metabolism, or decrease protein synthesis.

The nurse is caring for a client with an infected wound that is left to heal by secondary intention. Which of these observations does the nurse expect to make during assessment of the wound area? a) The wound is healing slowly with epithelial and scar tissues present. b) The scar tissue present is larger than the original wound. c) Sutures or staples are present in the wound. d) A large mass of scar tissue has developed at the original wound site.

The wound is healing slowly with epithelial and scar tissues present. Wounds healing by secondary intention undergo wound contraction resulting in a scar considerably smaller than the original wound. Cosmetically, this may be desirable because it reduces the size of the visible defect. Keloids result from abnormal wound healing, resulting in tumor-like masses caused by excess production of scar tissue. Sutures are present in wounds healing by primary intention.

A patient is admitted to the hospital with second degree burns over 30% of the body. Which is a characteristic of second degree burns? a) There is usually no blister formation. b) They involve the dermis and the epidermis. c) They involve the dermis only. d) They are dry.

They involve the dermis and the epidermis. Second degree burns involve the dermis and the epidermis. They are painful, moist, red, and blistered. Underneath the blisters is weeping, bright pink or red skin that is sensitive to temperature changes, air exposure, and touch.

A patient has sustained serious first degree, second degree, and third degree burns in a fire. Which best describes the burned area that involves the subcutaneous tissue, full thickness burn and involves muscle and bone? a) Third degree burn b) Second and third degree burns c) First and second degree burns d) Second degree burn

Third degree burn Third-degree full-thickness burns extend into the subcutaneous tissue and may involve muscle and bone. Thrombosed vessels can be seen under the burned skin, indicating that the underlying vasculature is involved. Third-degree burns vary in color from waxy white or yellow to tan, brown, deep red, or black. First-degree burns (superficial partial-thickness burns) involve only the outer layers of the epidermis. They are red or pink, dry,and painful. Second-degree burns involve both the epidermis and dermis. Second-degree partial-thickness burns involve the epidermis and various degrees of the dermis.

The community health nurse is teaching about prevention of hepatitis A. Which of these groups does the nurse suggest will benefit from this vaccine? a) Those working for the Centers for Disease Control and Prevention b) Homosexual women c) Those traveling to third world countries d) Those who have been recently exposed to hepatitis A

Those traveling to third world countries International hepatitis A vaccine is suggested for travelers to regions where sanitation is poor and endemic HAV infections are high, children living in communities with high rates of HAV infection, homosexually active men, and users of illicit drugs.

The nurse is caring for a client with a stab wound. The nurse recognizes that deficiency of which of these elements of the blood will interfere with hemostasis at the site of injury? a) Erythrocytes b) Thrombocytes c) Neutrophils d) Lymphocytes

Thrombocytes Platelets or thrombocytes are the cell fragments in the blood that are involved in primary hemostasis. Neutrophils and lymphocytes are white blood cells involved in inflammation, immune response, and wound healing. Erythrocytes carry oxygen to the tissues, but will not stop bleeding.

A patient is admitted with an exacerbation of Crohn's disease and has been experiencing a weight loss of 20 pounds in 2 weeks. The patient is unable to absorb nutrients from the intestine. The nurse would expect which of the following types of diet to be ordered for this patient? a) High-calorie, low-fat diet b) Low-residue diet c) High-fiber diet d) Total parenteral nutrition

Total parenteral nutrition Total parenteral nutrition consists of intravenous administration of hypertonic glucose solutions to which amino acids and fats may be added. This form of nutritional therapy may be needed when food cannot be absorbed from the intestine.

What is the length of time from infection with the AIDS virus to seroconversion? a) Immediately b) Approximately one week c) Up to six months d) 10 to 12 years

Up to six months The point at which an infected person converts from being negative for the presence of HIV antibodies in the blood to being positive is called seroconversion. Seroconversion typically occurs within 1 to 3 months after exposure to HIV but can take up to six months.

The nurse understands that the best way for a health care worker to protect against the transmission of HIV is to: a) use contact-isolation on all clients known to be HIV positive. b) use universal precautions on all clients. c) use proper hand-washing techniques before and after contact with all clients. d) use blood and body fluid precautions on clients known to be HIV positive.

Use universal precautions on all clients. Universal Blood and Body Fluid Precautions should be used in encounters with all people in the health care setting since it should be assumed that any person may have a transmissible infection. (less)

The nurse is teaching a patient with rheumatoid arthritis about pannus, which develops in the affected joint area. Which of the following does the nurse include to describe pannus? a) Vascular granulation tissue that destroys cartilage and bone b) Muscles in the area that atrophy from disuse c) Microorganisms that attack the joint space causing stiffness d) Reversible calcium deposits affecting the joints

Vascular granulation tissue that destroys cartilage and bone A network of new blood vessels in the synovial membrane that contributes to the advancement of the rheumatoid synovitis, called pannus, develops. This destructive vascular granulation tissue extends from the synovium to involve a region of unprotected bone at the junction between cartilage and the subchondral bone. Inflammatory cells found in the pannus have a destructive effect on adjacent cartilage and bone leading to reduced joint motion and the possibility of eventual ankylosis

In the vascular, hemodynamic stage of acute inflammation, initial vasoconstriction is followed by which of the following assessment findings? a) Pale hot skin b) Cool dry skin c) Bleeding. d) Vasodilation and congestion

Vasodilation and congestion Vasodilation allows more blood and fluid into the area of injury, resulting in congestion, redness, and warmth. A blood clot forms soon after.

A client with a long-standing diagnosis of human immunodeficiency virus (HIV) has recently developed neutropenia and been admitted to a hospital. Which of the following measures should be prioritized by the nurses who are providing his care? a) Administration of prophylactic antibiotics b) Administration of antiretroviral medications c) Supplementary oxygen and administration of bronchodilators d) Vigilant handwashing to protect against severe bacterial infections

Vigilant handwashing to protect against severe bacterial infections Neutropenia carries a greatly increased risk of infection that necessitates vigilant infection control measures and strict adherence to standard precautions such as handwashing. Antiretrovirals do not directly address his neutropenia, and antibiotics are not normally provided prophylactically. The client is susceptible to respiratory infections, but therapeutic interventions are not required until indicated by a diagnosis such as pneumonia. (

When caring for a postoperative client, in order to promote wound healing, which of these nutrients does the nurse encourage the client to consume? a) Vitamin C b) Vitamin D c) Vitamin E d) Magnesium

Vitamin C Vitamins are essential cofactors for the daily functions of the body; vitamins A and C play an essential role in the healing process. Vitamin C is needed for collagen synthesis. Vitamin K plays an indirect role in wound healing by preventing bleeding disorders that contribute to hematoma formation and subsequent infection. Proteins, fats, carbohydrates and microminerals such as zinc are required for wound healing as well.

A nursing student is assigned to care for a client who has AIDS. The student is unsure of what personal protective equipment should be worn while caring for this client. Which of the following statements is appropriate direction from the student's nursing instructor? a) No precautions are needed. b) Wear gloves if there is a chance you will come in contact with the client's blood or body fluids. c) Wear gloves, gown, and mask when in contact with the client. d) Have the client wear a mask when transporting him from one area of the hospital to another.

Wear gloves if there is a chance you will come in contact with the client's blood or body fluids. Universal Blood and Body Fluid Precautions should be used in encounters with all people in the health care setting. Occupational risk of infection for health care workers most often is associated with percutaneous inoculation of blood from a person with HIV infection. HIV infection is not transmitted through casual contact.

Which of the following procedures reduces the potential for infection primarily by addressing the portal of entry? a) Isolating patients who have antibiotic-resistant infections. b) Wiping down common areas with buffered bleach on a regular basis. c) Wearing gloves when contact with blood or body fluids is anticipated. d) Disposing of soiled clothing and bed linens in a dedicated receptacle.

Wearing gloves when contact with blood or body fluids is anticipated. The wearing of gloves specifically blocks the portal of entry to the health care worker through the use of a physical barrier. Bleaching and cleaning as well as disposing of soiled linen eliminate the source of infection by killing microorganisms, and isolating patients similarly address a source by minimizing contact with uninfected persons.

The nurse is caring for an obese client who has had abdominal surgery. The medical record states the wound has developed a dehiscence. Which of these does the nurse anticipate finding when changing the dressing? a) Wound edges are 1-1/2 inches apart. b) There is a foreign body in the wound. c) Wound has purulent drainage. d) The suture line is reddened.

Wound edges are 1-1/2 inches apart. Mechanical factors such as increased local pressure or torsion can cause wounds to pull apart, or dehisce.

Crohn's disease has a high incidence of fistula formation. Which of the following tests will the nurse expect to see for the diagnosis or presence of a fistula? a) Magnetic resonance imaging without contrast b) X-ray with contrast c) Rectal examination d) Stool specimen

X-ray with contrast Sigmoidoscopy is used for direct visualization of the affected areas and to obtain biopsies. Radiographic contrast studies provide a means for determining the extent of involvement of the small bowel and establishing the presence and nature of fistulas. Computed tomography scans may be used to detect an inflammatory mass or abscess.

Which of the following scenarios is most likely in the CSF of a patient with bacterial meningitis? a. CSF has a high neutrophil count and high protein count b. CSF has high neutrophil count and low protein count c. CSF has high glucose level and high red blood cell count d. CSF has high lymphocyte count and low red blood cell count

a. CSF has a high neutrophil count and high protein count

The hospitalized burn patient wants to know why you need to remove his dressings every day. It is painful and he wants to avoid uncovering his burn injury. You explain that removing the dressings promotes: a. Debridement b. Infection c. Skin function d. Drying the exudate

a. Debridement

Which of the following are considered antigen presenting cells? a. Dendritic cells b. Eosinophils c. B lymphocytes d. T lymphocytes

a. Dendritic cells

Immunodeficiency is the result of a. Failure of host defense mechanisms b. Hypersensitive immune responses c. Inappropriate immune response to self d. Immune response stimulated by antigens from other individuals

a. Failure of host defense mechanisms

A patient is diagnosed with achilles tendonitis. What would you expect to be the clinical manifestations of this condition given what you know about acute inflammation? a. Heel pain, swelling, limited range of motion b. Fever, anorexia, weakness c. Redness, inability to walk, ankylosis d. Fibrosis, degeneration, scarring

a. Heel pain, swelling, limited range of motion

What aspects of bacterial activity would be effective targets for pharmacologic treatment? a. Hepatitis B surface antigen (HBsAg) b. Hepatitis B core antigen (HBcAg) c. Prothrombin time (PTT) d. There is no way to distinguish the different forms of hepatitis B.

a. Hepatitis B surface antigen (HBsAg)

A major aspect of treatment for inflammatory conditions is: a. Ice, rest, elevation, compression b. Removal of the source of injury when possible c. Acetaminophen (Tylenol) d. Use of steroids

a. Ice, rest, elevation, compression

Which of the following best describes the inflammatory process in relation to an acute injury? a. Increased vascular permeability, dilation of vessels, and leukocyte attraction to the site of the injury b. Release of chemical mediators and vessel constriction c. Vessels dilate causing the release of chemical mediators d. None of the above

a. Increased vascular permeability, dilation of vessels, and leukocyte attraction to the site of the injury

During flu season, you get exposed to the influenza virus. Which component of your immune system will be the first to respond to this foreign pathogen? a. Innate b. Adaptive c. Humoral d. T-lymphocyte mediated

a. Innate

Which of the following is true regarding cytotoxic T lymphocytes? a. Recognize the MHC class I - antigen complex b. Recognize the MHC class II - antigen complex c. Carry the CD4 marker d. Present antigen to B lymphocytes for antibody production

a. Recognize the MHC class I - antigen complex

A wound is 6 cm × 6 cm × 4 cm. A wound with these dimensions will need to heal through: a. Secondary intention b. Primary intention c. Tertiary intention d. Scar tissue formation

a. Secondary intention

The preferred outcome from the inflammation process is: a. Tissue repair or regeneration b. Cellular alteration with scar tissue formation c. Metaplasia d. Cellular hypertrophy

a. Tissue repair or regeneration

A hypersensitivity reaction resulting from a yellow jacket sting is an example of a. Type I, immediate hypersensitivity reaction b. Type II, antibody-mediated reaction c. Type III, immune complex reaction d. Type IV, cell-mediated reaction

a. Type I, immediate hypersensitivity reaction

Your friend Sharilee is entering the nursing major at your college. As part of the admission requirements, she needs to have the hepatitis B vaccination. Sharilee asks you why she needs to have this vaccination. You tell her that it promotes the following type of immunity: A. Naturally acquired active immunity B. Naturally acquired passive immunity C. Artificially acquired active immunity D. Artificially acquired passive immunity

active immunity

During the inflammation process, a fever would be an example of: a. A local response to the inflammation process b. A systemic response to the inflammation process c. An indirect action d. A holistic response

b. A systemic response to the inflammation process

Which of the following conditions represents pathologic responses due to immunologic memory? a. Common cold b. Anaphylaxis c. Shingles d. Strep throat

b. Anaphylaxis

Depth of injury is important to determine about burns. You are in the sun too long without sunscreen and develop redness and blistering on your face, chest, and back. What depth of burn did you experience? a. Superficial partial-thickness burn b. Deep partial-thickness burn c. Full-thickness burn d. Dermal-thickness burn

b. Deep partial-thickness burn

You get a paper cut and experience pain at the site. This response is related to: a. Increased perfusion at the site b. Increased exudate and chemical mediators at the site c. Bacteria that have entered the wound d. Vasoconstriction at the site

b. Increased exudate and chemical mediators at the site

Treatment of altered immune response with corticosteroids is associated with which one of the following adverse effects? a. Decreased blood sugar b. Loss of bone mineral c. Thickening of skin d. Weight loss

b. Loss of bone mineral

A patient is diagnosed with ulcerative colitis. She has more than four bowel movements per day, but no fever, weakness, fatigue, or other systemic manifestations. Her disease would be classified as: a. Mild b. Moderate c. Severe d. In remission

b. Moderate

What is the most likely complication of an untreated UTI? a. Glomerulonephritis b. Pyelonephritits c. Fulminant UTI d. Urethritis

b. Pyelonephritits

Katelynn, a 13 year old female, is accompanied to the emergency department by her mother. Katelynn hit a tree with her four wheeler and has large abrasions on her right knee and forehead. Her mother asks if Katelynn will need stitches. You know that this type of wound will most likely heal by a. Primary intention b. Secondary intention c. Leukocyte adherence d. Third intention

b. Secondary intention

A white blood cell differential shows an increase in the number of monocytes and macrophages in the blood. This is typically means that: a. This is a new infection b. This is a chronic infection c. This is a viral infection d. The differential provides no useful information

b. This is a chronic infection

Acute inflammation occurs in response to a. Cellular repair b. Tissue injury c. Scar formation d. Cellular reproduction

b. Tissue injury

For the following season, you are concerned about getting the flu again. Which of the following statements is true? a. You continue to be at risk because nothing can protect you from reinfection. b. Vaccination for prevalent strains of influenza virus can provide improved protection against the disease. c. Premedication with immunosuppressants will provide protection against infection. d. Since you've had the flu once, you will be protected from getting it again.

b. Vaccination for prevalent strains of influenza virus can provide improved protection against the disease.

Immune suppression in AIDS is related to: a. Decreased platelet count b. Decreased red blood cell count c. Decreased lymphocyte count d. Elevated lymphocyte count

c. Decreased lymphocyte count

Which of the following may make a person more susceptible to getting an infection? a. Age between 4 and 46 years b. Experiencing a surgery that is healing by primary intention c. Final exams week d. A functioning immune system

c. Final exam week

A major difference between the acute and chronic inflammatory response is that in chronic inflammation: a. Inflammatory mediators are released b. Neutrophils are much more prominent c. Granulomas form around certain invaders d. Granulation tissue is present

c. Granulomas form around certain invaders

You know that the most important roles for chemical mediators are to: a. Release histamine b. Constrict vessels to control potential bacterial invasion in the area c. Induce vasodilation and increase capillary permeability d. Cause pain in order to decrease mobility

c. Induce vasodilation and increase capillary permeability

Which of the following is the most common cause of acute gastritis? a. Poor gastric perfusion b. Too much stomach acid c. Ingestion of aspirin, alcohol, or other chemicals d. H. pylori infection

c. Ingestion of aspirin, alcohol, or other chemicals

Which of the following markers identifies a nucleated body cell? a. CD4 b. BCR c. MHC I d. MHC II

c. MHC I

The transfer of secretory IgA from mother to infant during breast feeding is an example of a. Hypersensitivity reaction b. Active immunity c. Passive immunity d. Alloimmunity

c. Passive immunity

The inflammation process must prepare the site of injury for healing. This includes cleaning up the debris (dead and impaired tissue) at the site. This process of engulfing and digesting impaired tissue is called: a. Chemotaxis b. Diapedesis c. Phagocytosis d. Degranulation

c. Phagocytosis

The feeling that "something is not quite right" is considered which stage in infection? a. Point of infection with pathogen b. Incubation c. Prodrome d. Acute symptoms

c. Prodrome

A major difference between Crohn disease and ulcerative colitis is the presence of _____ in Crohn disease. a. A chronic inflammatory response b. Ulcers c. Skip lesions d. Histamine

c. Skip lesions

A hypersensitivity reaction resulting from complement activation due to insoluble antigen-antibody deposition is an example of a. Type I, immediate hypersensitivity reaction b. Type II, antibody-mediated reaction c. Type III, immune complex reaction d. Type IV, cell-mediated reaction

c. Type III, immune complex reaction

During the inflammation process, the vascular response triggers the following changes at the site of the injury except: a. Vasodilation b. Increased permeability of vessels c. Vasoconstriction d. Increased blood flow

c. Vasoconstriction

Which is not correct about an opportunistic infection? a) Chemotherapy or radiation can cause life threatening opportunistic diseases. b) An opportunistic infection primarily affects a host with a normal immunity. c) Fungi can cause life threatening opportunistic diseases. d) An opportunistic infection primarily affects an individual with a weakened immunity as a result of illness.

d) An opportunistic infection primarily affects an individual with a weakened immunity as a result of illness. An opportunistic infection primarily affects a host with weakened immunity as a result of illness, malnutrition, or medical therapy such as chemotherapy or radiation. Fungi can cause life-threatening opportunistic diseases.

Cells that play a role in phagocytosis include: a. Neutrophils b. Monocytes c. Macrophages d. All of the above

d. All of the above

Autoimmunity may be triggered by which one of the following? a. Elimination of self-reactive lymphocytes in central lymphoid tissues b. Persistent lymphocyte ignorance c. Impaired T-cell activation d. Close resemblance between foreign and self antigen

d. Close resemblance between foreign and self antigen

Katelynn asks what she can do to help the abrasions heal faster. All of the following are important factors in wound healing EXCEPT: a. Integrity of the vascular and cellular inflammatory responses b. Reformation of the extracellular matrix c. Regeneration of those cells capable of mitosis d. Decreased protein synthesis

d. Decreased protein synthesis

Which of the following is true regarding pharmacologic treatment for AIDS? a. Drugs are used to target increased red blood cell number b. Drugs are used to target increased white blood cell number c. Drugs are used to increase host DNA replication d. Drugs are used to inhibit HIV replication

d. Drugs are used to inhibit HIV replication

The pathology related to systemic lupus erythematosus is due to a. Neutrophil activation b. Delayed immunity c. Immunosuppression d. Immune complex deposition

d. Immune complex deposition

You are caring for Andrew, a 5 year old boy in the emergency department. He has a sliver in his right index finger that has been present for 2 days. Andrew refused to let his mother remove the sliver and now the tissue around the sliver is red, warm, and swollen. What is the most likely cause of the warmth and swelling? a. The release of prostglandins b. Cytokines c. Vasoconstriction in the microculation d. Increased blood flow to the area due to vasodilation

d. Increased blood flow to the area due to vasodilation

Which is not a local manifestation of acute inflammation? a. Edema b. Redness c. Loss of function d. Leukocytosis

d. Leukocytosis

You are looking to break the chain of infection by washing your hands frequently as you provide care for patients. which of the following links in the chain will be broken by this activity? a. Reservoir b. Host c. Portal of entry d. Mode of transmission

d. Mode of transmission

What is the one definitive test to diagnose rheumatoid arthritis? a. A positive rheumatoid factor (RF) b. An elevated erythrocyte sedimentation rate (ESR) c. A positive antinuclear antibody (ANA) d. One test is not definitive

d. One test is not definitive

Inflammation is ultimately needed to: a. Increase chemical mediators at the site to vasoconstrict the area b. Increase platelets at the site for clotting c. Restore functional cells d. Prepare the site for healing

d. Prepare the site for healing

Chronic inflammation differs from acute inflammation because it: a. Triggers the cellular response before the vascular response b. Occurs for a period of 1 week after the acute stage of inflammation c. Involves different cells d. Represents a persistent of recurrent state of inflammation lasting several weeks or longer.

d. Represents a persistent or recurrent state of inflammation lasting several weeks or longer.

Given the mode of transmission for influenza, how would you break the chain of infection and prevent spread? a. Administer antibiotics as directed b. Wash hands after toileting c. Disinfect table surfaces in the room d. Wear a mask

d. Wear a mask

In order for the cellular response to occur, which of the following processes need to take place? a. Cellular migration b. Cellular adherence c. Chemotaxis d. A and B e. A, B, and C

e. A, B, and C

The primary signs and symptoms of acute inflammation include all of the following except a. Redness and heat b. Swelling c. Pain and loss of function d. A and C e. All of the above

e. All of the above

Inflammation is a process that includes the following: a. Vascular and cellular responses b. The formation of exudates c. Preparation for tissue repair d. a and c e. a, b, and c

e. a, b, and c

When administering a corticosteroid to a patient with rheumatoid arthritis, a nurse includes which of the following in educating the patient about the medication? Select all that apply. a) "Corticosteroids are taken for life to reduce inflammation." b) "Pain is relieved with use of corticosteroid medications." c) "Corticosteroids do not prevent joint destruction." d) "Corticosteroids may be given to inhibit tumor necrosis factor." e) "These drugs are for short-term use."

• "Corticosteroids do not prevent joint destruction." • "Pain is relieved with use of corticosteroid medications." • "These drugs are for short-term use." Corticosteroids interrupt the inflammation and immune response at several levels, interfering with inflammatory cell adhesion and migration, impairing prostaglandin synthesis, and inhibiting neutrophil superoxide production. To avoid long-term side effects, they are used only for short-term therapy at a low dose level and should not be repeated more than a few times each year.

Which of the following manifestations would the nurse expect to observe in a female with pelvic inflammatory disease (PID)? Select all that apply. a) Temperature of 100°F (37.8°C) b) Purulent cervical discharge c) Elevated C-reactive protein d) White blood cell (WBC) count of 7500 cells/mL e) Lower abdominal pain

• Lower abdominal pain • Elevated C-reactive protein • Purulent cervical discharge Manifestations of PID include lower abdominal pain, dyspareunia, purulent cervical discharge, cervix pain with manual examination, temperature greater than 100°F, a WBC greater than 10,000 cells/mL, and elevated C-reactive protein.

The nurse knows that further teaching is needed when a client who is HIV positive makes which of the following statements? The client is aware that his blood work reflects a CD4+ cell count of 800. Select all that apply. a) "Since my CD4+ count is 800, I understand that I am of little risk of infecting my sexual partner." b) "I am disappointed that I am not yet cured of HIV." c) "I know I can still pass HIV to my sexual partner." d) "I am happy that my bloodwork indicates that the HAART therapy is effective."

• "I am disappointed that I am not yet cured of HIV." • "Since my CD4+ count is 800, I understand that I am of little risk of infecting my sexual partner." The normal range for the CD4+ T-cell count is 800-1000; however, there is no cure for HIV infection, and once infected, an individual is always at risk for transmitting HIV via sexual contact. The medications that are currently available to treat HIV infection decrease the amount of virus in the body, but they do not eradicate HIV.

A teenage patient is interested in determining the cause of acne. Which of the following factors contribute to an increase in sebum production that can lead to acne. Select all that apply. a) Inflammation of the sebaceous glands b) Enlargement of sebaceous glands c) Adolescence d) Rise in sex hormones levels e) Gender

• Adolescence • Enlargement of sebaceous glands • Inflammation of the sebaceous glands • Rise in sex hormones levels Sebum production is under the control of genetic and hormonal influences. Sebaceous glands are relatively small and inactive until people approach adolescence. The glands then enlarge, stimulated by the rise in sex hormones. Gland size directly influences the amount of sebum produced, and the level of androgens influences gland size. The sebaceous glands are the structures that become inflamed in acne.

A patient is admitted with having bloody emesis and pain. The diagnosis is acute gastritis. For which of the following causes of acute gastritis does the nurse assess the patient? Select all that apply. a) Acid reflux b) NSAIDs c) Aspirin d) Clostridium difficile e) Alcohol abuse

• Aspirin • NSAIDs • Alcohol abuse Acute gastritis is characterized by an acute mucosal inflammatory process, usually transient in nature. The erosive form is an important cause of acute gastrointestinal bleeding. The condition is most commonly associated with local irritants, such as aspirin, NSAIDs, alcohol, or bacterial toxins.

A nursing instructor is explaining how a pathogenic microorganism can invade the human body. The instructor knows that the students understand this process when they correctly identify which of the following ways the site of an infectious disease is determined? Select all that apply. a) By the portal of entry b) By the type of pathogen c) By the competence of the host's immunologic defense system d) By the number of pathogens e) By the age of the person

• By the type of pathogen • By the portal of entry • By the competence of the host's immunologic defense system The type of pathogen, the portal of entry, and the competence of the host's immunologic defense system ultimately determine the site of an infectious disease.

An adolescent is being assessed for her annual school physical. Which of the following describes acne vulgaris, which occurs most frequently in adolescence? Select all that apply. a) Rosacea b) Nodules c) Comedones d) Papulopustules

• Comedones • Papulopustules • Nodules Acne vulgaris occurs most frequently in adolescence and manifests as comedones, papulopustules, nodules. Rosacea is a chronic acneform disorder of middle-aged and older persons.

The nurse is admitting an older adult patient with an chronic wound. The nurse assesses for which of the following underlying conditions? a) Immobility b) Aging c) Diabetes d) Vascular disease

• Diabetes • Immobility • Vascular disease The nurse should assess for diabetes, immobility, and vascular disease as these are more often these are most often the cause of wounds in the older adult than aging.

The nurse is assessing a patient with rheumatoid arthritis. Which of the following would the nurse expect to find? Select all that apply. a) Swelling b) Erythema c) Warmth d) Crepitus e) Tenderness

• Erythema • Swelling • Warmth • Tenderness RA has symptoms that apply to the affected joints showing signs of inflammation, swelling, tenderness, warmth and possibly reduced motion. Crepitus is related to degenerative disease.

The nurse is caring for a patient with rheumatoid arthritis (RA). Which of the following assessment findings does the nurse expect? (Select all that apply.) a) Fatigue b) Weight loss c) Flushed skin d) Increased appetite e) Anorexia

• Fatigue • Weight loss • Anorexia RA is characterized by weight loss, generalized aching, anorexia, fatigue, as well as joint changes such as pain and stiffness. Increased appetite and flushed skin are not symptoms of RA.

The basement membrane surrounding a patient's foot wound remains intact, a fact that bodes well for the wound-healing process. Which of the following components constitute this form of the extracellular matrix (ECM)? Select all that apply. a) Prostaglandins b) Glycoproteins c) Lymphocytes d) Water-hydrated gels e) Fibrous structural proteins

• Fibrous structural proteins • Water-hydrated gels • Glycoproteins There are three basic components of ECM: fibrous structural proteins (e.g., collagen and elastin fibers), water-hydrated gels (e.g., proteoglycans and hyaluronic acid) that permit resilience and lubrication, and adhesive glycoproteins (e.g., fibronectin, laminin) that connect the matrix elements one to another and to cells.

Every one in the family except the mother currently has influenza. What should the nurse tell the mother are appropriate measures to avoid this infection? Select all that apply. a) Avoid close contact with family members b) Immunization c) Sterilization d) Frequent hand washing

• Immunization • Frequent hand washing • Avoid close contact with family members Immunization, frequent hand washing, and avoiding close contact with family members are appropriate measures to avoid influenza. Sterilization is not an appropriate measure for influenza.

Which of the following assessment findings will help the nurse to confirm the diagnosis of rheumatoid arthritis? Select all that apply. a) Wrist pain b) Limited joint mobility c) Finger and hand pain d) Asymmetrical joint involvement e) Joint stiffness

• Joint stiffness • Finger and hand pain • Wrist pain • Limited joint mobility Joint stiffness and limited joint mobility, usually starting in the fingers, hands, and wrists are hallmarks of rheumatoid arthritis. Joint involvement is usually symmetrical not asymmetrical.

A patient is admitted to the hospital with first and second degree burns. Which assessment findings are associated with first degree burns? Select all that apply. a) Full thickness burn b) Painful c) Red or pink d) Wet

• Red or pink • Painful First-degree burns (superficial partial thickness burns) involve only the outer layers of the epidermis. They are red or pink, dry, and painful.

A nurse is teaching a patient about psoriasis and the nurse knows that teaching is effective when the client correctly identifies which of the following lesions as being associated with psoriasis? Select all that apply. a) Pustules b) Plaques c) Cysts d) Nodules e) Scales

• Scales • Plaques A plaque is an elevated, palpable, solid mass with a circumscribed border. Flakes secondary to desquamated, dead epithelium that may adhere to kin surface; color varies (silvery, white); texture varies (thick, fine). Both types of skin lesions can be found with psoriasis.

A physician is attempting a differential diagnosis of a 30 year-old female who is suspected of having systemic lupus erythematosus (SLE). Which of the following aspects of the physician's assessment and the client's history would be considered potentially indicative of SLE? Select all that apply. a) The client has been hospitalized twice in the past for pleural effusions. b) Blood work indicates low red cells, white cells and platelets. c) She complains of intermittent joint pain. d) The woman states that she has numerous environmental allergies. e) The client has a "butterfly rash" on her nose and cheeks.

• She complains of intermittent joint pain. • The client has been hospitalized twice in the past for pleural effusions. • Blood work indicates low red cells, white cells and platelets. • The client has a "butterfly rash" on her nose and cheeks. A butterfly rash, joint pain, pleural effusion and low levels of blood cellular components are all associated with SLE. Environmental allergies are not noted to be risk factors or associated symptoms of the disease.

A 68 year-old woman has had her mobility and quality of life severely affected by rheumatoid arthritis (RA). Place the following pathophysiological events involved in her health problem in the correct order that they most likely occurred. Use all the options. a) Interaction between rheumatoid factor (RF) and IgG b) T-cell mediated immune response c) Destruction of articular cartilage d) Inflammatory response e) Pannus invasion

• T-cell mediated immune response • Destruction of articular cartilage • Inflammatory response • Pannus invasion • Interaction between rheumatoid factor (RF) and IgG RA is thought to begin with a T-cell mediated immune response which precipitates interaction between IgG and RF that constitutes an immune response. Pannus invasion is one consequence of this interaction, the ultimate result of which is destruction of cartilage


Kaugnay na mga set ng pag-aaral

ENVIRONMENTAL SCIENCE EXAM 4 CHAPTER 17

View Set

SOC-1113-22269 Exam 1 study guide

View Set

Unit 12 body system 4:The Male Reproductive System

View Set

Chapter 9 Class Test - Legal Descriptions

View Set

Chapter 22 - Performance and Breach of Sales and Lease Contracts

View Set

Business 150 Module 3 Study Guide

View Set

Chapter 20 Biology, Chapter 21 Biology, Chapter 22 Biology

View Set

GOVT 2306 Chapter 3 Review: The Texas Legislature

View Set